✅ DDx: Dyspnea, Mediastinum Flashcards

1
Q

👨🏿‍⚕️ Physical Exam

A

VITALS BP, HR rhythm and quality, RR 120/80 85 12 T 37*C, volume depletion [For “the list” looking for: temp, respiratory rate range, urine output [L (2.8)/weight (Kg)/hr], record each drain

GEN ASSESSMENT – “Ill or well appearing?” level of distress, nutritional status, Skin, Hair, Nails for color, lesions and moisture

HEENT

Head

Inspection: head and scalp, SAD

Eyes

Inspection: The eyes should be examined for scleral icterus and the skin for jaundice. Inspection of orbital area, conjunctiva, sclera, iris, pupils (edema, lesions, cemetery, discharge, shape)Pupillary reaction to light (consensual)

Ears

TM mobility: (Pneumatic insuflator): When pressure in the middle ear space is equivalent to ambient air pressure, the normal TM moves laterally and medially with a pressure pulse from the bulb as low as 10 to 15 mm H2O.

Reduced TM mobility is caused by fluid, a solid mass in the middle ear space, retraction, atrophy, or sclerosis.

Perforation also causes the TM to become immobile, although this may be obscured by otorrhea.

Hearing: Whisper, webber, rhine

Mouth

Inspection: oral cavity, lips, teeth, gums (color, exudates, lesions, tonsil size), lymph nodes (submandibular, anterior cervical, supraclavicular), palpate thyroid (size, symmetry, nodules, enlargement, tenderness)

Neck

Trachea (pleural effusion pneumothorax)

sinuses, TMJ

CARDIO

Palpate precordium, PMI (lifts, thrills, heaves)

Auscultation at 2nd intercostal space, RSB, LSB, fourth and fifth ICS LSB apex, bell;

Pulses – R, PT, DP, C, B, F, P

Capillary refill

Edema

Cyanosis

Documentation: cor (heart) nsr (normal sinus rhythm), -m, r, g

PULMONARY

Inspection of chest wall, ribs (SAD), accessory muscles, retractions, respiratory distress (tachypnea, hyperpnia, paradoxical breathing, pursed lips, cyclical breeding patterns [kussumal/cheyene stokes], possible skeletal abnormalities scoliosis kyphosis, pectus excuvatum),

Percussion (fluid or solid tissue in lung or pleural space, dullness to percussion {lobar pneumonia}, resonance, pneumothorax, emphysema)

  • Normal lung - Resonnance
  • Consolidation - Dullness
  • ♒ Effusion - Dullness
  • Pneumothorax - hyperrossonance
  • Atelectasis - Dullness

Hyperresonance to percussion is characteristic of an air-filled thoracic cavity (eg, pneumothorax) or hyperinflated lung tissue (eg, emphysema).

Auscultation (vesicular, wheezes, asthma, bronchitis, bronchial breath sounds {pneumonia}, rales rhonchi mucous obstruction).

  • Normal lung - Bronchovesicular (hilar),
  • Consolidation - Increased
  • Hemothorax - decreased?
  • ♒ Pleural effusion - Decreased or absent
  • Pneumothorax - Decreased or absent
  • Atelectasis - Decreased or absent

When a portion of the lung is consolidated (eg, lobar pneumonia), the density of tissue/fluid increases and dullness to percussion is detected. In addition, sound conducts more rapidly through the consolidated lung, resulting in increased intensity of breath sounds and a more prominent expiratory component. More rapid sound conduction also results in increased tactile fremitus as well as egophony (sounds like the letter “A” when the patient says the letter “E”) in areas of lung consolidation. Crackles are also often heard.

Rales = fine crackles

Ronchi = Coarse

Vesicular = normal

Tubular = Bronchial

The best areas to listen for right middle lobe findings would be (1) the right anterior midclavicular line between the fifth and
sixth ribs
and (2) the rightmidaxillary line between the fourth and sixth ribs.The right middle lobe is not heard posteriorly, and thelung examination is incomplete if
the physician does not listen anteriorly or medially.

SpecialTests Stromal notch angle costosternal tactile fremitus 99 E Toy boat

Tactile fremitus: if fluid (pleural effusion) or air (pneumothorax) are present just outside the lung in the thoracic cavity, they can act to insulate sound and vibration originating from the lung, which leads to decreased breath sounds and decreased tactile fremitus.

Decreased or absent when vibrations from the larynx to the chest surface are impeded by chronic obstructive pulmonary disease, obstruction, pleural ♒ effusion, or pneumothorax.

Increased: PNA (consolidation)

ABDOMEN

Inspection (drape) general appearance and level of comfort or discomfort – lesions, scars, dilated veins, echymoses, muscle separation,

Auscultation - bowel sounds, (clicks gurgles - detection of ileus/obstruction)

Percussion peritonitis/ascites/hepatosplenomegaly

Palpation – light, deep, liver edge (start by examining the quadrant of the abdomen where the patient is experiencing the least pain - Guarding is typically absent with deeper sources of pain such as renal colic and pancreatitis)

Documentation: “Abd nondistended, +bs, +tender RLQ; rectal guaiac -nl tone, -mass -

How well did you know this?
1
Not at all
2
3
4
5
Perfectly
2
Q

Intravenous fluidsFluids

A

Tonicity

Isotonic

  • 0.9% (normal) saline, Lactated Ringer solution
    • Volume resuscitation (eg, hypovolemia, shock)
  • Albumin (5% or 25%)
    • Volume replacement, treatment of SBP or HRS

Hypotonic

  • Dextrose 5% in water, 0.45% (half-normal) saline
    • Free water deficit
  • Dextrose 5% in 0.45% (half-normal) saline [Dextrose 5% in water (initially slightly hypotonic) & dextrose 5% in 0.45% saline (initially hypertonic) become markedly hypotonic due to metabolism of glucose.]
    • Maintenance hydration

Hypertonic

  • 3% (hypertonic) saline
    • Severe, symptomatic hyponatremia
How well did you know this?
1
Not at all
2
3
4
5
Perfectly
3
Q

Volume Depletion

A

Children are at risk for intravascular volume depletion due to the high frequency of gastroenteritis, a higher surface area-to-volume ratio resulting in increased insensible losses, and an inability to access fluids themselves or communicate their needs. In gastroenteritis, volume depletion occurs when the extracellular losses exceed the fluid intake. As a result, oral or intravenous fluid therapy is required in order to replenish the normal intravascular volume.

The initial step in managing children with dehydration is to determine its severity. The ideal method of assessing dehydration is to determine the measured change in weight because 1 kg of acute weight loss equals 1 L of fluid loss. A child’s weight, however, changes constantly, making it difficult to obtain an accurate recent “well” weight. Therefore, the degree of dehydration often has to be determined by the clinical history and physical examination, which can be divided into the following categories:

Mild dehydration (3-5% volume loss) presents with a history of decreased intake or increased fluid loss with minimal or no clinical symptoms.

Moderate dehydration (6-9% volume loss) presents with decreased skin turgor, dry mucus membranes, tachycardia, irritability, a delayed capillary refill (2-3 seconds), and decreased urine output.

Severe dehydration (10-15% volume loss) presents with cool, clammy skin, a delayed capillary refill (>3 seconds), cracked lips, dry mucous membranes, sunken eyes, sunken fontanelle (if still present), tachycardia, lethargy, and minimal or no urine output. Patients can present with hypotension and signs of shock when severely dehydrated.

Oral rehydration therapy should be the initial treatment in children with mild to moderate dehydration. Children with moderate to severe dehydration (which is the category that this patient is in) should be immediately resuscitated with intravenous fluids to restore perfusion and prevent end organ damage. Isotonic crystalloid is the only crystalloid solution recommended for intravenous fluid resuscitation in children, which explains why isotonic saline is the best answer of the choices given.

How well did you know this?
1
Not at all
2
3
4
5
Perfectly
4
Q

♣ Clubbing

A

Digital clubbing describes bulbous enlargement and broadening of the fingertips due to connective tissue proliferation at the nail bed and distal phalanx. It is diagnosed when the angle between the nail fold and the nail plate is >180° (Lovibond angle). Clubbing can occur by itself or associated with hypertrophic osteoarthropathy, which presents with painful joint enlargement, periostosis of long bones, and synovial effusions.

Clubbing may be hereditary, but is most often due to pulmonary or cardiovascular diseases. The most common causes of secondary clubbing are lung malignancies 🦀, cystic fibrosis, and right-to-left cardiac shunts.

Pathophysiology involves megakaryocytes that skip the normal route of fragmentation within pulmonary circulation (due to circulatory disruption from tumors, chronic lung inflammation) to enter systemic circulation.

Megakaryocytes become entrapped in the distal fingertips due to their large size and release platelet-derived growth factor (PDGF) and vascular endothelial growth factor (VEGF). PDGF and VEGF have growth-promoting properties that increase connective tissue hypertrophy and capillary permeability and vascularity, ultimately leading to clubbing.

Hypertrophic pulmonary osteoarthropathy (HPOA) is a subset of HOA where the clubbing and arthropathy are attributable to underlying lung disease like lung cancer, tuberculosis, bronchiectasis, or emphysema.

How well did you know this?
1
Not at all
2
3
4
5
Perfectly
5
Q

Acute “Dyspnea”:

“Subjective SOB”

A

Anaphylaxis

Asthma (may also present as chronic dyspnea)

Pneumonia

Pneumothorax

Pleural effusion/hemothorax

Pulmonary embolism

Aspiration

How well did you know this?
1
Not at all
2
3
4
5
Perfectly
6
Q

Anaphylaxis

A

Allergic reaction

Triggers

  • Food (eg, nuts, shellfish)
  • Medications (eg, β-lactam antibiotics)
  • Insect stings

Clinical manifestations

  • Cardiovascular
    • Vasodilation → hypotension & tissue edema
    • Tachycardia
  • Respiratory
    • Upper airway edema → stridor & hoarseness
    • Bronchospasm → wheezing
  • Cutaneous
    • Urticarial rash, pruritus, flushing
  • Gastrointestinal
    • Nausea, vomiting, abdominal pain

Treatment

  • Intramuscular epinephrine
  • Airway management & volume resuscitation
  • Adjunctive therapy (eg, antihistamines, glucocorticoids)

The most common diagnostic criteria for anaphylaxis include an acute illness involving the skin/mucosa AND either respiratory or cardiovascular compromise (hypotension or signs of end-organ hypoperfusion). Other common manifestations include gastrointestinal, neurologic, and ocular symptoms.

Dx: In patients who have been exposed to a known allergen, a significant decrease in blood pressure (eg, >30% below the patient’s baseline) is sufficient to make the diagnosis. If the diagnosis is unclear, serum tryptase or plasma histamine may be helpful.

The result of a widespread response to antigen, leading to massive release of histamine and other substances from mast cells and basophils.

Hx: Bronchoconstriction leads to wheezing and dyspnea, and upper airway edema may lead to asphyxiation.

Urticaria, facial edema, wheezing

Hx: Multiple food allergies.

Ddx: Includes anaphylaxis versus allergic reaction. In the setting of the diarrhea in conjunction with the rash most likely anaphylactic. Would recommend use of the epipen if the rash appears in the setting of continued diarrhea. Patient given 1ml benadryl which is likely below the dose needed.

Tx: Treatment involves supportive care including airway management and medications such as:

  • 🥇 IM Epinephrine is a beta-2 and alpha-1 adrenergic receptor agonist, which causes bronchial smooth muscle relaxation (eg, decreases wheezing) and vasoconstriction (eg, decreases edema, increases blood pressure), respectively. Some cases require additional doses of epinephrine for refractory symptoms
  • Benadryl dosed at 1 mg/kg (0.25 mg/kg) (5 mg/kg/day)(maximum daily 300)
  • PO Decadron (dexamethasone) 🌚
  • Topical hydrocortisone if the rash does not completely resolve

Latex can lead to a spectrum of IgE-mediated type 1 hypersensitivity reactions, including isolated urticarial skin rash (eg, erythema, wheals), rhinoconjunctivitis, and anaphylaxis.

Cx: Medications such as nonsteroidal anti-inflammatory drugs or beta-adrenergic blockers can exacerbate anaphylaxis by resulting in nonimmunologic mast cell activation or unopposed alpha-adrenergic effects, respectively.

How well did you know this?
1
Not at all
2
3
4
5
Perfectly
7
Q

ARDS

A

An inflammatory condition that can develop in the setting of infection (eg, sepsis, pneumonia), trauma, or other conditions (eg, massive transfusion, pancreatitis). Lung injury causes the release of proteins, inflammatory cytokines, and neutrophils into the alveolar space. This leads to leakage of bloody and proteinaceous fluid into the alveoli, alveolar collapse due to loss of surfactant, and diffuse alveolar damage. As a result:

Gas exchange is impaired due to ventilation-perfusion mismatch

Lung compliance (ability to expand) is decreased (stiff lungs) due to both loss of surfactant and increased elastic recoil of edematous lungs

Pulmonary arterial pressure is increased (pulmonary hypertension) due to hypoxic vasoconstriction, destruction of lung parenchyma, and compression of vascular structures from positive airway pressure in mechanically ventilated patients

Findings suggestive of ARDS include respiratory distress, diffuse crackles on lung examination, severe hypoxemia, and bilateral alveolar infiltrates on chest imaging, which occur within a week of an insult. The partial pressure of arterial oxygen (PaO2) decreases and leads to an increased fraction of inspired oxygen (FiO2) requirement. As a result, PaO2/FiO2 (P/F) is decreased (<300 mm Hg) with lower P/F ratios indicating more severe degrees of ARDS.

Tx:

Mechanical ventilation: improves oxygenation by providing an increased fraction of inspired oxygen (FiO2) and positive end-expiratory pressure (PEEP) to prevent alveolar collapse. The goal is to maintain arterial partial pressure of oxygen (PaO2) at 55-80 mm Hg, which roughly corresponds to oxygen saturations > 88% Immediately following intubation, a high FiO2 (eg, >60%, or 0.6) is usually provided, and ventilator settings can subsequently be adjusted based on the results of the first arterial blood gas analysis.

Barotrauma: Avoiding complications of mechanical ventilation by using lung-protective strategies such as low tidal volume ventilation (LTVV): LTVV (6 mL/kg of ideal body weight) decreases the likelihood of overdistending alveoli and provoking barotrauma due to high plateau pressures (pressure applied to small airways and alveoli).

ICU: ARDS is a term used to describe a constellation of clinical and radiographic signs and symptoms reflecting pulmonary edema in the absence of elevated pulmonary venous pressures. ARDS is relatively common in the ICU population and is associated with high mortality (50%). The syndrome results from a variety of causes, including sepsis or pulmonary infection, severe trauma, and aspiration of gastric contents, all of which together account for 80% of cases. Whatever the initial cause, all share activation of the complement pathway with damage to the alveolar capillary endothelium, increased vascular permeability, and subsequent development of first interstitial and then alveolar pulmonary edema. Clinically, there is severe respiratory distress characterized by marked hypoxia that responds poorly even to administration of high concentrations of oxygen. Pulmonary capillary wedge pressure is usually normal. Decreased surfactant production leads to poor lung compliance and atelectasis that results in an intrapulmonary shunt with perfusion but no effective ventilation. Positive End Expiratory Pressure (PEEP) can help to decrease atelectasis, shunting while improving oxygenation. Cx: Patients surviving the syndrome may progress to pulmonary fibrosis or have no sequelae. The longer and more severe the ARDS, the more likely are long term consequences. Other factors may also contribute, such as age and preexisting COPD.

How well did you know this?
1
Not at all
2
3
4
5
Perfectly
8
Q

Hypoxemia

A

Reduced PiO2 (High altitude) Normal A-a; corrects with O2

Hypoventilation (CNS depression, morbid obesity) Ventilation is decreased, diffusion and V/Q matching are intact; therefore, the A-a gradient is normal and increased FiO2 increases gas exchange and improves hypoxemia. Causes of alveolar hypoventilation and respiratory acidosis include the following:

  • Pulmonary/thoracic diseases: Chronic obstructive pulmonary disease, obstructive sleep apnea, obesity hypoventilation, scoliosis
  • Neuromuscular diseases: Myasthenia gravis, Lambert-Eaton syndrome, Guillain-Barré syndrome
  • Drug-induced hypoventilation: Anesthetics, narcotics, sedatives
  • Primary central nervous system dysfunction: Brainstem lesion, infection, stroke

Diffusion limitation (Emphysema, ILD); Increased A-a ⬇

In the normal lung of an upright patient, V and Q are highest in the bases of the lung as gravity creates hydrostatic pressure acting on both air and blood.

V/Q mismatch (caused by localized dead-space ventilation and/or intrapulmonary shunting). (Small PE, lobar pneumonia, atelectasis)

Hypoxemia due to localized intrapulmonary shunting typically does correct with increased FiO2 because only a portion of the lungs is affected and the normally ventilated alveoli compensate via increased O2 transfer.

Large intrapulmonary shunt (Diffuse pulmonary edema); Does not correct with O2 ⬇

Because diffuse pulmonary edema prevents air from reaching the alveoli throughout much of the lungs (eg, >50%), an increase in the fraction of inspired O2 (FiO2) does not correct the hypoxemia.

Increased alveolar-arterial (A-a) gradient is another characteristic of pulmonary edema, as with any process causing V/Q mismatch or impaired diffusion across the A-a membrane. Finally, pulmonary edema also leads to fluid collection within the lung interstitium, resulting in stiffening of the lungs (decreased lung compliance).

Large dead-space ventilation (Massive PE, right-to-left intracardiac shunt)

Although alveolar ventilation is normal, there is no perfusion of large portions of the lung (extreme V/Q mismatch), resulting in minimal gas exchange.

PAO2 = (FiO2 x [Patm – PH2O]) – (PaCO2/R)

A-a gradient = PAO2 – PaO2

How well did you know this?
1
Not at all
2
3
4
5
Perfectly
9
Q

Hemic Hypoxia

A

Some medications - most commonly topical anesthetics (eg, benzocaine), dapsone, and nitrates (in infants) - cause the iron component of hemoglobin to be oxidized, thereby forming methemoglobin, which cannot bind oxygen. The remaining normal hemoglobin also has an increased affinity for oxygen, resulting in less oxygen delivery to tissues. Because methemoglobin absorbs light at distinct wavelengths, pulse oximetry commonly is ~85% (as seen in this patient) regardless of the true oxygen saturation. In parallel, blood gas analysis frequently returns a falsely elevated result for oxygen saturation (eg, 99% in this patient) as it provides an estimate based only on the PaO2, not on effective hemoglobin-oxygen binding. These inaccurate readings create the large oxygen saturation gap.

Cyanosis can occur when methemoglobin comprises ~10% of total hemoglobin, but hypoxia symptoms (eg, headache, lethargy) occur only when levels surpass 20%. At levels >50%, there is risk of severe symptoms (eg, altered mental status, seizures, respiratory depression) and death. Treatment involves discontinuing the causative agent and administering methylene blue, which helps reduce iron to its normal state.

How well did you know this?
1
Not at all
2
3
4
5
Perfectly
10
Q

💨 Aspiration Syndromes

A

Follows gravitational flow of aspirated contents

Hx: Impaired consciousness, post anesthesia, common in alcoholics, debilitated, demented pts; anaerobic (Bacteroides and Fusobacterium). Observed aspiration, symptoms start during or shortly after eating or vomiting. Patient with altered mental status or abnormal gag reflex at baseline. Reduced consciousness, neuromuscular disorders, and intratracheal or intraesophageal devices all are factors which may predispose patients to aspiration by compromising the patient’s airway defense mechanisms.The effects of aspiration are determined by volume of the aspirate and the nature of the aspirated material. These determine the extent and severity of any inflammatory response. Chemical irritants may be acid, alkali or paticulate in nature depending on gastric contents.

Supine: The common locations of pneumonia are the posterior segment of the (R) upper lobe and superior segment of the (R) lower lobe. These three segments are often referred to as the aspiration segments of the lung.

Upright: The basilar segments of the lower lobes of both lungs are susceptible to aspiration if the patient aspirates while erect or sitting up.

Unilateral, and sometimes bilateral, crackles, more commonly on the right, fever

Cx: Aspiration of gastric acid is also known as Mendelson’s Syndrome, it is the most common type of aspiration. which leads to a chemical “pneumonitis” and potentially acute respiratory distress syndrome (ARDS) due to extensive desquamation of the bronchial epithelium with subsequent pulmonary edema. The degree of irritation to the lung is directly dependent on the acidity and volume of the aspirated fluid. The lung responds to pH < 2.5 with severe bronchospasm and the release of inflammatory mediators. The initial result is a chemical pulmonary edema. Secondary infection may or may not result. The clinical manifestations occur within minutes of the event and include cough, dyspnea, wheezing and diffuse crackles. Fever and an elevated white count will occur in the majority of patients. The consequences of aspiration range from shock to uncomplicated resolution of the initial event. The chest film in patients that progress to pneumonitis will reveal pulmonary consolidation within the first two days. The consolidation is usually perihilar and bilateral, though asymmetric. The radiographic findings begin to stablize or resolve by the third day. Some patients’ radiographs will show worsening of the consolidation as well as findings associated with pneumonia, including pleural effusions and abscess formation. Aspiration may also cause ARDS.

Tx: There is no evidence to support the use of antibiotics or high-dose steroids. Treatment consists of supplemental oxygen and other supportive measures.

How well did you know this?
1
Not at all
2
3
4
5
Perfectly
11
Q

Asthma

A

Hx: Episodic cough triggered by cold air and hyperventilation. The symptoms are suggestive of cough-variant asthma. Occurs in larger airways.

Mild persistent asthma: Symptoms more than 2 days per week but not daily, and wakes up once a week but not nightly.

Dx: The most important component in the diagnosis of asthma is history.

Spirometry: Abnormal spirometry results (reversible obstruction) can help to confirm an asthma diagnosis, but normal results do not exclude asthma. A reduced FEV1 or a reduced FEV1/FVC ratio documents airflow obstruction. An increase in FEV1 of >12% with a minimum increase of 200 mL in FEV1 after bronchodilator use establishes the presence of airflow reversibility and the diagnosis of asthma.

Peak flow variability: A patient with normal spirometry results but marked diurnal variability (based on a peak-flow diary kept for >2 wk) may have asthma, which may warrant an empiric trial of asthma medications or bronchoprovocation testing.

Bronchoprovocation testing: In a patient with a history highly suggestive of asthma and normal baseline spirometry results, a low PC20 (concentration of inhaled methacholine needed to cause a 20% drop in FEV1) on methacholine challenge testing supports a diagnosis of asthma. A normal bronchoprovocation test essentially excludes asthma. [Should be used cautiously, as life-threatening bronchospasm may occur].

Chest radiography: Chest radiography may be needed to exclude other diagnoses but is not recommended as a routine test in the initial evaluation of asthma.

Allergy skin testing: There is a strong association between allergen sensitization, exposure, and asthma. Allergy testing is the only reliable way to detect the presence of specific IgE to allergens. Skin testing (or in vitro testing) may be indicated to guide the management of asthma in selected patients, but results are not useful in establishing the diagnosis of asthma.

Tx: Regardless of disease severity, all patients are prescribed a short-acting, inhaled β-agonist medication. Intermittent asthma

If short-acting bronchodilators are needed for symptom relief more than twice a week for daytime symptoms or twice a month for nighttime awakenings, a long-acting controller medication is indicated. Use of more than one canister of short-acting β-agonist per month may be a clue to poor control of asthma and warrants further investigation.

1 SABA: ALBUTEROL PRN

Mild persistent asthma: is treated with a single long-term controller medication. A low-dose inhaled glucocorticoid is the preferred long-term controller medication; alternatives include a mast cell stabilizer, leukotriene modifier, or sustained-release methylxanthine.

2 LOW DOSE INHALED 🌑 CORTICOSTEROID: PRED, METHYLPRED, BUDESONIDE,

ALTERNATIVE:

MAST CELL STABILIZER: NEDOCROMIL, CROMOLYN

LEUKOTRIENE MODIFIER: MONTELEUKAST 10, ZAFIRLUKAST, ZILEUTON

MONOCLONAL (IgE) ANTIBODY: OMALIZUMAB, BENRALIZUMAB (FASENRA®)

[COPD] DUONEB®/COMBIVENT®: IPATROPIUM AND ALBUTEROL $ (SHORT ACTING?)

[COPD] SPIRIVA®: TIOTROPIUM $$$

Moderate persistent asthma: is treated with one or two long-term controller medications. Use either low doses of inhaled glucocorticoid and a long-acting β-agonist (preferred) or medium doses of a single inhaled glucocorticoid. In patients who remain symptomatic while taking medium doses of inhaled glucocorticoids, the addition of a long-acting bronchodilator (eg, salmeterol) results in improved lung physiology, decreased use of short-acting β-agonists, and reduced symptoms when compared with doubling the dose of inhaled glucocorticoid.

LOW DOSE INHALED CORTICOSTEROID: PRED, METHYLPRED, BUDESONIDE ➕

3 LABA: SALMETEROL (SEREVENT), FORMETEROL

OR

MEDIUM DOSE INHALED CORTICOSTEROID: PRED, METHYLPRED, BUDESONIDE

ALTERNATIVE:

LOW DOSE INHALED CORTICOSTEROID: PRED, METHYLPRED, BUDESONIDE +

LEUKOTRIENE MODIFIER: MONTELEUKAST 10 (SINGULAIR®), ZAFIRLUKAST, ZILEUTON

SYMBICORT®: BUDESONIDE AND FORMETEROL (COPD) “LAMA BEFORE LABA”

ADVAIR®: FLUTICASONE AND SALMETEROL

Severe persistent asthma: may require at least three daily medications to manage their disease (ie, high doses of an inhaled glucocorticoid plus a long-acting bronchodilator and possibly oral glucocorticoids).

MEDIUM DOSE INHALED CORTICOSTEROID: PRED, METHYLPRED, BUDESONIDE + LABA: SALMETEROL (SEREVENT®), FORMETEROL AND MONOCLONAL (IgE) ANTIBODY: OMALIZUMAB (XOLAIR®), BENRALIZUMAB (FASENRA®) IN THOSE WITH ALLERGIES

HIGH DOSE INHALED CORTICOSTEROID: PRED, METHYLPRED, BUDESONIDE + LABA: SALMETEROL (SEREVENT), FORMETEROL + ORAL CORTICOSTEROID AND MONOCLONAL (IgE) ANTIBODY: OMALIZUMAB (XOLAIR®), BENRALIZUMAB (FASENRA®) IN THOSE WITH ALLERGIES

Cx:

Allergic bronchopulmonary aspergillosis (ABPA) is almost assured if the first six of these seven criteria are present: a history of asthma, peripheral eosinophilia, elevated serum IgE levels, skin reactivity to Aspergillus antigen, precipitating antibodies to Aspergillus antigen, a chest radiograph showing transient or fixed infiltrates, and central bronchiectasis.

If the asthma attack is severe, the patient will develop a pulsus paradoxus (an inspiratory drop in systolic blood pressure of more than 10 mm Hg).

Acute asthma exacerbation: The initial physiologic response to an acute pulmonary insult is an increase in respiratory drive and resultant hyperventilation. This response may be caused by a combination of hypoxemia, anxiety due to a sensation of dyspnea, and signals from thoracic neural receptors influenced by change in lung volume and presence of inflammatory chemicals (eg, prostaglandins, histamines). Hyperventilation results in a decrease in the PaCO2 and a primary respiratory alkalosis, which is the typical presentation in an acute asthma exacerbation. ❗Normal pH and PaCO2, which in the setting of ongoing increased work of breathing indicates an inability to maintain adequate ventilation. This inability to meet the demands of increased respiratory drive is caused by respiratory muscle fatigue or severe air trapping and suggests impending respiratory collapse.

Aspirin is a common trigger for bronchoconstriction in patients with asthma, especially those with concurrent chronic rhinitis and nasal polyps.

Nonselective beta blockers (eg, propranolol, nadolol, sotalol, timolol) act on β1 and β2 receptors and often trigger bronchoconstriction in patients with underlying asthma.

Cardioselective beta blockers (eg, metoprolol, atenolol, bisoprolol, nebivolol) act predominantly on β1 receptors and are generally considered safe in patients with mild-to-moderate asthma. However, all beta blockers can trigger bronchoconstriction, especially when administered in large doses.

How well did you know this?
1
Not at all
2
3
4
5
Perfectly
12
Q

🦠 Pneumonia

A

A space occupying lesion without volume loss; caused by bacteria, viruses, mycoplasmae and fungi.

Ddx: Airspace filling not distinguishable radiographically: fluid (inflammatory), cells (cancer), protein (alveolar proteinosis) and blood (pulmonary hemorrhage).

Dx: The x-ray findings of pneumonia are airspace opacity, lobar consolidation, or interstitial opacities. There is usually considerable overlap.

Lobar - classically Pneumococcal pneumonia, entire lobe consolidated and air bronchograms common. Abrupt in onset, with fever, pleuritic chest pain, and purulent sputum production.

An “Air bronchogram” is a tubular outline of an airway made visible by filling of the surrounding alveoli by fluid or inflammatory exudates.

Ddx: air bronchograms: Lung consolidation (PNA), pulmonary edema, nonobstructive pulmonary atelectasis, severe interstitial disease, neoplasm, and normal expiration.

Lobular - often Staphlococcus, multifocal, patchy, sometimes without air bronchograms

Interstitial - Viral or Mycoplasma; latter starts perihilar and can become confluent and/or patchy as disease progresses, no air bronchograms. “Ground Glass” is a radiology descriptive term (used in both chest radiographs and CT imaging) to indicate that blood vessels are not obscured as would be the case in alveolar lung opacities. Both atypical bacterial and viral organisms may produce pneumonias that differ radiographically from more common bacteria such as pneumococcus. They may produce a ground glass appearance and increased interstitial markings. The CXR appearance of Pneumocystis pneumonia is typically bilateral, diffuse interstitial (“reticular”) or ground glass opacities.

Hx: “Atypical pneumonia” due to C pneumoniae or 🏒M. pneumoniae: Patients often complain of a sore throat at the beginning of the illness and a protracted course of symptoms. Physical examination is often unimpressive compared to radiograph findings and the diagnosis is often not made as the course is often indistinguishable from other lower respiratory infections.

💨Aspiration - follows gravitational flow of aspirated contents; impaired consciousness, post anesthesia, common in alcoholics, debilitated, demented pts; anaerobic (Bacteroides and Fusobacterium). In a supine patient who has aspirated, the common locations of pneumonia are the posterior segment of the upper lobe and superior segment of the lower lobe. The superior segment of the right lower lobe is the segment most likely to develop aspiration pneumonia.

Diffuse pulmonary infections - Community acquired (Mycoplasma, resolves spontaneoulsy) nosocomial immunocompromised host (bacterial, fungal, PCP)

🏥 Nosocomial by definition occur 3 days after admission. Patients in the ICU are often relatively immunocompromised secondary to their primary disease and are subject to iatrogenic factors which increase their sucseptabilty to pneumonia-causing pathogens. These include the following: endotracheal tubes, which defeat many patient defense mechanisms; medications used to reduce gastric acid, which may promote bacterial growth in the stomach; and the use of antibiotics, which may selectively encourage the growth of some pathogenic bacteria. Nosocomial pneumonias are often polymicrobial and caused by 🔮gram-negative enteric pathogens. The offending organisms often include Pseudomonas species, E-coli, Klebsiella species, and Proteus species (Pseudomonas, debilitated, mechanical vent pts, high mortality rate, patchy opacities, cavitation, ill-defined nodular).

🌆 Community-acquired pneumonias, which usually are caused by 🏮 gram-positive species

Dx:

The radiographic appearance of pneumonia may be difficult to differentiate from atelectasis or early ARDS. Classically, pneumonia first appears as patchy opacifications or ill-defined nodules. It is often multifocal and bilateral, occurring most often in the gravity dependent areas of the lung. This feature makes it difficult to distinguish from atelectasis or pulmonary edem. E-coli and pseudomonas species can rapidly involve the entire lung. Their symmetric pattern often simulates pulmonary edema. The presence of patchy air space opacities, air bronchograms, ill-defined segmental consolidation or associated pleural effusion support the diagnosis of pneumonia. Occassionally, in gram-negative pneumonias small luciencies may be found within consolidated lung which may represent unaffected acini or areas of air trapping. This is particularly likely to occur in patients with underlying COPD. However, these must be distinguished from lucencies created by cavitation and abscess formation.

Cx:

Pleural effusions caused by gram-negative organisms are more likely to represent empyema and therefore require drainage.

Lung abscess formation and bronchopleural fistulas.

In consolidative pneumonia, the alveoli become filled with inflammatory exudate, leading to marked impairment of alveolar ventilation in that portion of the lung. The result is right-to-left intrapulmonary shunting, which describes perfusion of lung tissue in the absence of alveolar ventilation, an extreme form of ventilation/perfusion (V/Q) mismatch (V≈0). A characteristic of intrapulmonary shunting is inability to correct hypoxemia with increased concentration of inspired oxygen (FiO2). Other causes of V/Q mismatch (eg, emphysema, interstitial lung disease, pulmonary embolism) allow for correction of hypoxemia with an increase in FiO2 because V>0. In practice, increased FiO2 typically leads to some improvement in hypoxemia in patients with pneumonia because only a portion of the lung is being affected by intrapulmonary shunting.

Secondary bacterial pneumonia is the most common complication of influenza but is rare in young individuals (most are age >65). An exception occurs with community-associated methicillin-resistant Staphylococcus aureus (CA-MRSA), an organism that preferentially attacks young patients with influenza. CA-MRSA causes severe, necrotizing pneumonia that is rapidly progressive and often fatal. Manifestations include high fever, productive cough with hemoptysis, leukopenia, and multilobar cavitary infiltrates. Most patients require admission to the intensive care unit and broad-spectrum, empiric antibiotics, including either vancomycin or linezolid.

Recurrent pneumonia occurring in the same anatomic location of the lung raises suspicion for localized airway obstruction, which, if present, can lead to impaired bacterial clearance and predisposition to infection (eg, postobstructive pneumonia). Potential causes of localized airway obstruction include:

  • External bronchial compression due to lymphadenopathy, expanding neoplasm, or vascular anomaly
  • Internal bronchial obstruction due to a foreign body, bronchiectasis, bronchial stenosis, or, rarely, endobronchial carcinoid
  • Lung malignancy is a potential cause of localized airway obstruction and may present with episodes of recurrent pneumonia. CT scan of the chest should be used to evaluate patients in whom there is suspicion for lung malignancy.
How well did you know this?
1
Not at all
2
3
4
5
Perfectly
13
Q

💥 Pneumothorax

A

Spontaneous pneumothorax

  • Primary: no preceding event or lung disease; often thin, young men
  • Secondary: underlying lung disease (eg, COPD, CF)

Spontaneous - idiopathic, asthma, COPD, pulmonary infection, neoplasm, Marfan’s syndrome, and smoking cocaine.

Secondary spontaneous pneumothorax: Cigarette smoking markedly increases the risk of pneumothorax. Chronic destruction of alveolar sacs leads to the formation of large alveolar blebs, which can eventually rupture and leak air into the pleural space.

Signs & symptoms

  • Chest pain, dyspnea
  • ↓Breath sounds, ↓chest movement
  • Hyperresonant to percussion

Imaging

  • Visceral pleural line
  • Absent lung markings beyond pleural edge

Management

  • Small (≤2 cm): observation & oxygen administration
  • Large & stable: needle aspiration or chest tube

Tension pneumothorax

  • Life-threatening
  • Often due to trauma or mechanical ventilation

Signs & symptoms

  • Same as spontaneous plus:
  • Hemodynamic instability
  • Tracheal deviation away from affected side

Imaging

  • Same as spontaneous plus:
  • Contralateral mediastinal shift
  • Ipsilateral hemidiaphragm flattening

Management

  • Urgent needle decompression or chest tube placement

Defined as air inside the thoracic cavity but outside the lung (pleural space).

Iatrogenic (most common) - Caused by a physician during surgery or central line placement. Pneumothorax is a common complication of invasive procedures such as central line placement, especially in the mechanically ventilated patient. Barotrauma also can lead to pneumothorax, complicating the intubated patient’s medical course. The air may also arrive at the intrapleural space by rupture of alveoli (blebs), extension of a pneumomediastinum, or communication with extrathoracic air following trauma or surgery.

Trauma - motor vehicle accident is another important cause.

Tension Pneumo - TP develops when accumulated air (due to injured lung tissue) causes high intrathoracic pressurethat compresses the vena cava and impedes cardiac venous return, resulting in decreased cardiac output and hypotension. This occurs when a one-way valve has formed, allowing air to flow into the pleural space during inspiration but trapping it during expiration. Eventually the pressure buildup is large enough to collapse the lung and shift the mediastinum away from the tension PTX.

Px: Trachea deviating away from the side of the traumatized lung. This occurs secondary to trauma or during mechanical ventilation. Breath sounds will be faint or distant, percussion will be hyperresonant, and fremitus will be decreased.

Dx: Mediastinal shift is usually seen in a tension pneumothorax, but the use of PEEP may prevent this from occurring. The most reliable sign of tension pneumothorax is depression of a hemidiaphragm. Other signs of tension pneumothorax include shifting of the heart border, the superior vena cava, and the inferior vena cava. The shifting of these structures can lead to decreased venous return.

Tx: When TP is suspected, decompression (eg, needle thoracostomy) should be performed immediately to prevent cardiovascular collapse. Needle thoracostomy can be performed quickly and should precede intubation. This is an important exception to the typical order of establishing the airway first (ie, airway, breathing, circulation) but is necessary because positive-pressure ventilation (eg, intubation and mechanical ventilation) rapidly increases accumulated air and intrathoracic pressure, exacerbating TP and causing cardiovascular collapse. Following needle decompression, tube thoracostomy is required for definitive pneumothorax management.

Hydropneumothorax is both air and fluid in the pleural space. It is characterized by an air-fluid level on an upright or decubitus film in a patient with a pneumothorax. Some causes of a hydropneumothorax are trauma, thoracentesis, surgery, ruptured esophagus, and empyema.

Primary pneumothorax: affects tall, thin men and may be recurrent. It is thought to be due to the rupture of subpleural blebs in response to high negative intrapleural pressures.

Px: Hypotension, unilateral chest expansion, decreased fremitus, hyperresonance,
and diminished breath sounds.

Dx:

CRX appears as air without lung markings in the least dependant part of the chest. Generally, the air is found peripheral to the white line of the pleura. In an upright film this is most likely seen in the apices. A PTX is best demonstrated by an expiration film. It can be difficult to see when the patient is in a supine position. In this position, air rises to the medial aspect of the lung and may be seen as a lucency along the mediastinum. It may also collect in the inferior sulci causing a deep sulcus sign.

Apicolateral - Appears as a thin, white pleural line with no lung markings beyond. The presence of lung markings beyond this line, though, does not exclude pneumothorax. This is especially true in the patient with parenchymal disease which may alter the compliance of affected lobes, making their collapse more difficult to detect radiographically. Parenchymal disease may also make visualization of the pleural line more difficult or impossible.

Supine patient - In the supine patient, intrapleural air rises anteriorly and medially, often making the diagnosis of pneumothorax difficult. The anteromedial and subpulmonary locations are the initial areas of air collection in the supine patient. An apical pneumothorax in a supine patient is a sign that a large volume of air is present. Subpulmonic pneumothorax occurs when air accumulates between the base of the lung and the diaphragm. Anterolateral air may increase the radiolucency at the costophrenic sulcus. This is called the deep sulcus sign.

Subpulmonary - Occasionally, a posterior subpulmonary pneumothorax will result in visualization of the more superior anterior diaphragmatic surface and the inferior posterior diaphragmatic surface, resulting in the double-diaphragm sign.

Other signs of subpulmonic pneumothorax include a hyperlucent upper quadrant with visualization of the superior surface of the diaphragm and visualization of the inferior vena cava.

Anteromedial - Anteromedial pneumothoraces are differentiated into those which are superior or inferior to the pulmonary hilum. A superior anteromedial pneumothorax may result in visualization of the superior vena cava or azygos vein on the right. An inferior anteromedial pneumothorax may be evidenced by delineation of the heart border and a lucent cardiophrenic sulcus. This is the key sign of a pneumothorax as this is the highest point in the supine patient, where the air will accumulate first.

How well did you know this?
1
Not at all
2
3
4
5
Perfectly
14
Q

Pleural Effusion

A

Pleural effusions form when fluid accumulates in the pleural space between the parietal and visceral linings of the lung.

These fluids include blood, chyme, pus, transudates or exudates. In the ICU patient, pleural effusions are extremely common. In patients on medical services the most common cause is congestive heart failure, while up to two-thirds of patitient will develop pleural effusions following upper abdominal surgery. Patients undergoing thoracotomy or median sternotomy will also usually develop pleural effusions. Other causes of fluid accumulation in the intrapleural space include pulmonary embolism, neoplastic disease, subphrenic inflammatory processes (e.g. pancreatitis), pneumonia, trauma, and ascites.

Pleural effusions of less than 1 centimeter in maximal depth when evaluated by bedside ultrasound should not be sampled by thoracentesis.

In general, the probe should be placed between the mid-scapular line and the posterior axillary line between the 8th and 9th ribs.

Cx (Thoracentesis): Pneumothorax, bleeding, re-expansion pulmonary edema, diaphragmatic injury, splenic or liver injury, soft tiddue infection or empyema.

🚌 Transudative pleural effusions: Caused by alterations in hydrostatic or oncotic pressures with normal capillary permeability. The relatively low pleural fluid protein value means that capillary permeability is normal and that only small molecules (ie, salt and water) can leak out.

💔 Heart failure: The elevated pressure from left ventricular end diastole and the left atrium transmits back to the alveolar capillaries to increase hydrostatic pressure. 🎡Diuresis can increase pleural fluid protein and lactate dehydrogenase, resulting in discordant exudate (25% of effusions can meet exudative criteria if the patient has received aggressive diuretics prior to thoracentesis); loss of hydrostatic pressure. b/l

💨 Atelectasis: Small effusion caused by increased negative intrapleural pressure; common in patients in the intensive care unit.

🔥 Constrictive pericarditis: b/l effusions with normal heart size; jugular venous distention present in 95% of cases.

⚪ Duropleural fistula: Cerebrospinal fluid (CSF) in the pleural space; caused by trauma and surgery.

Extravascular migration of central venous catheter: With saline or dextrose infusion.

🐄 Hepatohydrothorax: Occur due to small defects in the diaphragm. These defects permit peritoneal fluid to pass into the pleural space, which occurs much more commonly on the right side due to the less muscular hemidiaphragm.

Hypoalbuminemia: Small bilateral effusions; edema fluid rarely isolated to pleural space; gastrosis nephrosis, cirrhosis.

Nephrotic syndrome: Typically small and bilateral effusions; unilateral effusion with chest pain suggests pulmonary embolism; loss of oncotic pressure.

📺 Peritoneal dialysis: Small bilateral effusions common; rarely, large right effusion develops within 72 h of initiating dialysis.

ESRD and volume overload

🔵Superior vena cava obstruction: Acute systemic venous hypertension.

Trapped lung: Unexpandable lung; unilateral effusion as a result of imbalance in hydrostatic pressures from remote inflammation.

💛 Urinothorax: Unilateral effusion caused by ipsilateral obstructive uropathy

🔰 Chylous effusion:

🐄 Chirrhosis

🧠 CNS leak from trauma or VP shunt

Exudative Pleural Effusions: are the result of an inflammatory process causing proteins to leak across the capillary membrane; an inflammatory (or neoplastic) process allows large molecules to enter the pleural space.

💡 “Light’s criteria”

🦠 Infectious (bacterial, 💜 TB, fungal, parasitic)

Parapneumonic Effusion:

Occur in up to 50% of patients who are admitted to the hospital with bacterial pneumonia.

Tend to be small, free-flowing, sterile, and resolve with antibiotics (uncomplicated). The presence of loculated (non-free-flowing) fluid predicts a poor response to treatment with antibiotics alone.

If bacteria persistently cross into the pleural space, patients can develop complicated parapneumonic effusions or empyemas. Approximately 10% become complicated or progress to empyema.

Empyema: Bacterial infection in the pleural space, is suggested by the presence of a loculated effusion on upright and decubitus chest radiography or by obvious loculation on chest CT. Dx: Characterized by a very high white cell count, “turbid” fluid, and pH less than 7.2.

Dx: Pleural 🍭glucose <30 mg/dL in particular suggests an empyema or rheumatic effusion.)

🦀 Malignancy (carcinoma, lymphoma): A massive effusion, occupying the entire hemithorax, increases the likelihood of an underlying lung cancer or cancer involving the pleura (metastatic, mesothelioma).

Bilateral exudative effusions suggest malignancy but also occur in patients with pleuritis due to systemic lupus erythematosus and other collagen vascular diseases.

🦋 Collagen Vascular Disease: Bilateral exudative effusions suggest malignancy but also occur in patients with pleuritis due to systemic lupus erythematosus (SLE) and other collagen vascular diseases (RA).

🔴 PE: small pleural effusions due to hemorrhage or inflammation. The effusions tend to be exudative and grossly bloody, and they can be associated with pain due to pleural irritation.

Inflammatory (🧽pancreatitis, ARDS, ☢ radiation, sarcoid, post-CABG, pos-🔪 surgical)

Peripancreatic effusions simply occur in response to nearby inflammation of the pancreas (so-called sympathetic effusion). Occasionally, a pancreaticopleural fistula will form, leading to an exudate with very high amylase level. Such effusions often require chest tube drainage. Almost all effusions resulting from pancreatitis are left-sided exudates.

🤮 Boerhaave syndrome: Gastric contents enter the left pleural space and cause an inflammatory (exudative) effusion w/ very low pH

💥 Trauma (hemothorax, thoracic duct injury)

Px: Chest examination of a pleural effusion reveals distant or absent breath sounds, a pleural friction rub, decreased tactile fremitus, and flatness to percussion as the fluid in the thoracic cavity acts to insulate sound and vibration originating from the lung. A pleural friction rub is a raspy, grating sound heard in both inspiration and expiration due to inflamed surfaces rubbing against each other.

Dx:

Upright film - Costophrenic angle blunting and decreased visibility of the lower lobe vessels are commonly the result of pleural fluid pooling.

Blunting on the lateral and if large enough, the posterior costophrenic sulci (“meniscus sign”). Sometimes a depression of the involved diaphragm will occur. A large effusion can lead to a mediastinal shift away from the effusion and opacify the hemothorax. In the supine film, an effusion will appear as a graded haze that is denser at the base. The vascular shadows can usually be seen through the effusion. An effusion in the supine view can veil the lung tissue, thicken fissure lines, and if large, cause a fluid cap over the apex.

Approximately 200 ml of fluid are needed to detect an effusion in the frontal film

50-75 ml of fluid must collect before costophrenic blunting is visible in the erect patient

Lateral decubitis film - Helpful in confirming an effusion in a bedridden patient as the fluid will layer out on the affected side (unless the fluid is loculated). The lateral decubitus position can also differentiate between loculated and free effusions.

>500 ml of fluid must accumulate before you expect to see changes in the supine patient’s chest x-ray

Loculations occur when the visceral and parietal pleura become partially adherent. Tx: They may require guided placement of chest tubes for adequate drainage.

Lateral film: approximately 75ml must accumulate before you expect to see changes

Ultrasound - also a key component in the diagnosis. Ultrasound is also used to guide diagnostic aspiration of small effusions.

Subpulmonic Effusions:

Up to a liter of fluid may collect between the diaphragm and the lung without blunting of the costophrenic angle. Radiographically, subpulmonic effusions appear as a raised diaphragm with flattening and lateral displacement of the dome. The gastric bubble and splenic flexture of the colon show displacement inferiorly. The distance between the lung and the stomach bubble will exceed 2 cm in subpulmonary effusions. The lateral decubitus film can usually resolve any question of the presence of a subpulmonary effusion.

Interlobar Effusions:

The diagnosis of interlobar effusion can often be challenging, especially in the presence of incomplete pleural fissures. A CT may be required to make the diagnosis. Another challenge can be differentiating between a loculated effusion in the minor fissure and right middle lobe atelectasis. An effusion appears as an homogenous density with biconvex edges and preservation of the minor fissure, while atelectasis appears as an inhomogenous density with concave margins and obliteration of both the right heart border and minor fissure.

📈 Pleural Fluid Laboratory Studies:

🧪 ⚪ Protein

💡 Lights Criteria are used to determine whether a pleural effusion
is from a transudative or exudative process. Setting a low bar (by requiring only one of the three tests to be positive) increases the sensitivity of detection for exudative processes but lowers the specificity.

Ratio of:

“Pleural fluid” ⚪ protein“fluid comes first”toserum ⚪ protein

TpF/TPS> 0️⃣.5️⃣✋🏿

Exudate

Pleural fluid 🥛 LDH to serum LDH:

LDHF/LDHS >0️⃣.6️⃣✋🏿👆🏿

Exudate

🥛 LDHF > 2/3 upper limit of normal for serum

Transudate if ALL negative

🧪 Cell count with diff:

⚪Leukocyte count:

<1000/µL 🚲 Transudative

>10,000/µL 🚗(10 x 109/L): 🦠parapneumonic effusion (a noninfected effusion occurring in the pleural space adjacent to a bacterial pneumonia); 🧽acute pancreatitis; splenic infarction; and subphrenic, hepatic, and splenic abscesses.

>50,000/µL 🚙(50 x 109/L): complicated parapneumonic effusion or 🦠empyema (a parapneumonic effusion with persistent bacterial invasion) and empyema (established infection with pus in the pleural space) but occasionally occurs with acute pancreatitis and pulmonary infarction.

🦀Malignant disease and 💜tuberculosis typically present as a lymphocyte-predominant exudate. Additionally, although transudates may be blood-tinged, a grossly bloody effusion may be associated with cancer, tuberculosis, or trauma.

Neutrophils: >50%: 🦠 parapneumonic effusion, 🔴pulmonary embolism, abdominal disease.

Lymphocytes: >80%: 💜 tuberculosis (most common), 🦀malignancy (lymphoma), coronary artery bypass surgery, rheumatoid pleuritis, sarcoidosis.

🔴Erythrocyte count: >100,000/µL 🏡

: 🦀malignancy, trauma (hemothroax), 🦠parapneumonic effusion, 🔴pulmonary embolism

🧪 pH (send on ice): For patients with effusions associated with a pulmonary infection or with a malignancy, the pH has clinical significance.

✅ Normal pleural fluid pH is 7.60 to 7.66 🔵

Transudates are usually due to systemic factors (eg, increased hydrostatic pressure or hypoalbuminemia) and associated with a pleural fluid pH of 7.45 to 7.55 🔵

Exudates are usually due to inflammation and range from 7.30 to 7.45 🔴 Patients with a malignant effusion and a low pH have a much higher yield for finding malignant cells on cytology. They also tend to have a shorter survival.

Pleural fluid pH <7.20 is usually due to increased acid production by pleural fluid cells and bacteria (eg, empyema) or decreased hydrogen ion efflux from the pleural space (eg, pleuritis, tumor, pleural fibrosis); the most common causes are complicated parapneumonic effusion or 🦠 empyema, 💜tuberculous pleurisy, esophageal rupture, rheumatoid pleuritis, and 🦀malignancy. Patients
with a pulmonary infection and a pleural effusion with pH less than 7.20 are considered to have an empyema–effectively an abscess in the pleural space–and require chest tube placement and/or surgical drainage.

🧪 🍭Glucose: The glucose level of the pleural fluid is significant because it helps to narrow the potential causes of exudative effusions..

Pleural fluid glucose <60 mg/dL is usually due to rheumatoid pleurisy, complicated parapneumonic effusion (empyema), 🦀malignant effusion, 💜tuberculous pleurisy, lupus pleuritis, or esophageal rupture. Decreased in 🦠empyema is due to the high metabolic activity of leukocytes (and/or bacteria) in the fluid. Pleural glucose <30 mg/dL in particular suggests an empyema or rheumatic effusion.)

🧪 Adenosine Deaminase:

>40 U/L: 💜tuberculosis (>90%), complicated parapneumonic effusion (30%) or 🦠empyema (60%), malignancy (5%)

🧪 Cytology: Positive: 🦀malignancy (metastatic)

🧪 Gram stain and Culture:

Positive: infection

🧫Gram stain + culture

  • Fungi
  • Bacteria
  • TB

Hematocrit fluid to blood ratio: ≥0.5: hemothorax

🧪 Amylase:

Pleural fluid amylase should be measured only when 🧽pancreatic disease, esophageal rupture, or 🦀malignancy is considered.

🧪 Triglycerides:

🔰A chylous effusion (milky white fluid) is highly likely if the serum triglyceride level is 💯>110 mg/dL. A chylous effusion (chylothorax) is commonly caused by leakage of lymph, rich in triglycerides, from the thoracic duct due to 💥 trauma or obstruction (eg, 🦀lymphoma). Most are left-sided. 🧽 Pancreatitis?

Cholesterol crystals: Rheumatoid Arthritis; The fluid is usually 🔰 greenish-yellow in color

Tx: Unexplained effusions larger than👆🏿 1️⃣cm should be aspirated.

💉Thoracentesis is not necessary in patients who have small pleural effusions (<1 cm between the lung and chest wall on lateral chest radiograph) associated with heart failure, pneumonia, or heart surgery.

Septations = loculated -> thoracostomy (+/- tPa) -> thoracotomy may be necessary

CHF - Diuresis -> thoracentesis if fails

Caution is advised when considering performing a thoracentesis in patients with severe coagulopathy, thrombocytopenia, hemodynamic compromise, or on mechanical ventilation. Pneumothorax is the major complication of thoracentesis.

How well did you know this?
1
Not at all
2
3
4
5
Perfectly
15
Q

Pulmonary embolism / DVT

A

The pathophysiology of pulmonary embolism consists of both hemodynamic and respiratory embarrassment. Approximately 90% of pulmonary embolisms are the result of venous thrombosis in the lower extremities. Hemodynamic consequences occur when more than half the cross sectional area of the pulmonary vascular bed is occluded. Cx: This situation leads to pulmonary hypertension and in the acute setting right heart failure. Increased alveolar dead space (a result of ventilated but underperfused lung) leads to hypoxemia and respiratory failure. Pulmonary infarction is a rare consequence of pulmonary embolism in patients without concommitent compromise of the bronchial circulation. Approximately 10% of patients with PE have occlusion of a peripheral pulmonary artery by thrombus, causing pulmonary infarction. These small peripheral thrombi are more likely to cause pleuritic chest pain and hemoptysis, due to inflammation and irritation of the lung parenchyma and adjacent visceral and parietal pleura. Generally, infarctions are hemorrhagic and located in the lower lobes.

Hx: Symptoms of dyspnea, tachypnea, hemoptysis, hypoxemia, and pleuritic chest pain have been attributed to pulmonary embolism but are neither sensitive nor specific. Indeed, the most valuable indicators of pulmonary embolism are a history of risk factors and or a previous embolic event. Many different medical and surgical conditions are associated with increased risk of pulmonary embolization, including immobilization, trauma, surgery, shock, obesity, pregnancy, polycythemia vera, and antithrombin-III deficiency.

Sudden-onset dyspnea, nonproductive cough, tachycardia, and mild hypoxia is highly suggestive of acute pulmonary embolism (PE).

Risk factors for venous thromboembolism (VTE) are either inherited (eg, Factor V Leiden, prothrombin gene mutation, protein C deficiency) or acquired (eg, immobilization, surgery, malignancy, medications).

Tx: The first step in managing patients with suspected pulmonary embolism (PE) is supportive care (eg, oxygen, intravenous fluids for hypotension). The next step is assessing absolute contraindications to anticoagulation (eg, active bleeding, hemorrhagic stroke). Patients with contraindications should undergo diagnostic testing for PE, with appropriate treatment (eg, inferior vena cava filter) if positive. Patients without contraindications can be assessed with the modified Wells criteria for PE pretest probability. In patients in whom PE is unlikely based on these criteria, diagnostic testing is performed before anticoagulation is considered. However, anticoagulation (eg, low-molecular-weight heparin or unfractionated heparin) should be given prior to diagnostic testing in patients with likely PE, especially when patients are in moderate to severe distress.

Modified Wells

+3 points

Clinical signs of DVT (leg swelling)

Alternate diagnosis less likely than PE

+1.5 points

Previous PE or DVT

Heart rate >100

Recent surgery or immobilization (in last 4 weeks)

+1 point

Hemoptysis

Cancer (treated within the last 6 months)

Total score for clinical probability
≤4 = PE unlikely
>4 = PE likely

Dx: Patients with a score >2 (eg, pitting edema, calf swelling >3 cm compared to the other leg) are more likely to have DVT.

The workup of suspected PE can be divided into two populations.

Compression ultrasonography is the preferred initial test as it can be performed quickly in most emergency departments. If the patient has leg swelling, a venous ultrasound of the leg veins should be done to exclude DVT.

Inpatient:

CT pulmonary angiogram (CTPA) will likely be more definitive than a V/Q scan, as it may disclose other causes of hypoxia not shown on CXR.

Outpatient:

V/Q scan should be the first test and will less likely be indeterminate than in the inpatient setting. There is also a lower radiation dose for V/Q scans than for CTPA. Ventilation-perfusion scans detect abnormalities of blood flow in comparison to the pattern of ventilation, with areas of mismatch between perfusion and ventilation being evidence of vascular occlusion due to a pulmonary embolus.

Pulmonary arteriogram is the definitive, but more invasive test if these studies are inconclusive a

Clinically stablity (normotensive, mild hypoxemia) with no evidence of distress, the diagnosis of PE can be confirmed with:

CT angiography (CTA)[Helical CT?]: If CTA confirms PE, clinical judgment can dictate whether anticoagulation is initiated or other options are pursued (eg, inferior vena cava filter placement) based on the estimated risk of bleeding from the peptic ulcer. If CTA confirms PE, clinical judgment can dictate whether anticoagulation is initiated or other options are pursued (eg, inferior vena cava filter placement) based on the estimated risk of bleeding.

D-dimer assay is a simple, relatively noninvasive test that has been shown to have a high negative predictive value, especially if suspicion for DVT is low. [also increased in MI, stroke, Ischemia, A-fib, DIC, infection, trauma…]

💀CRX shows tachycardia (only 10% S1Q3T3)

Due to its relative lack of sensitivity, the chest x-ray in patients with suspected pulmonary embolism is usually relegated to the role of ruling out other disorders which may have a similar clinical presentation. The chest x-ray is also very useful when interpreting ventilation-perfusion scans. Though the majority of patients with pulmonary embolism in retrospect do have abnormalities on the chest x-ray, findings are usually too non-specific to be of diagnostic value. Without infarction there are few chest film signs of pulmonary emboli. These include: discoid atelectasis, elevation of the hemidiaphragm, enlargement of the main pulmonary artery into what has been described as the shape of a “sausage” or a “knuckle” (Palla’s sign), and pulmonary oligemia beyond the point of occlusion (Westermark’s sign).

Massive PE is defined as PE complicated by hypotension and/or acute right heart strain.

Cx: Occasionally, pulmonary embolisms will cause infarction causing a unique constellation of radiographic signs. Multifocal consolidation of the affected lung may occur in 12 to 24 hours following the embolic event. A consolidation which begins at the pleural surface and is rounded centrally is called a Hamptom’s Hump. These types of consolidation differ from pneumonia in that they lack air bronchograms. Up to 50% of patients with pulmonary embolism will also have ipsilateral or bilateral pulmonary effusions, although these are certainly nonspecific findings. Nevertheless, it is unusual for pulmonary infarctions to be diagnosed by chest radiography although infarctions are known to occur much more frequently. Presumably infarcts are confused with or indistinguishable from atelectasis or pneumonia. Despite the low sensitivity of these signs, the chest radiograph remains an important first step in the diagnosis of pulmonary embolism, primarily to exclude other causes of hypoxemia and to aid in the interpretation of the ventilation/perfusion scan.

Westermark’s sign (oligemia in area of involvement), increased size of a hilum (caused by thrombus impaction), atelectasis with elevation of hemidiaphragm and linear or disk shaped densities, pleural effusion, consolidation, and Hampton’s hump (rounded opacity). In the case of pulmonary infarctions, the main radiographic feature is multifocal consolidation at the pleural base in the lower lungs.

Tx: Early, effective anticoagulation decreases the mortality risk of acute PE and should be considered in patients without absolute contraindications (eg, hemorrhagic stroke, massive gastrointestinal bleed). Treatment with unfractionated heparin, low-molecular-weight heparin, fondaparinux, or therapeutic doses of warfarin.

Intravenous or subcutaneous unfractionated heparin, low-molecular-weight heparin, or fondaparinux. Most patients with pulmonary embolism are treated in the hospital, although carefully selected, stable patients may be candidates for outpatient treatment. Following initial therapy, patients are usually transitioned to warfarin for long-term therapy, with factor Xa and direct thrombin inhibitors being increasingly-available options for this purpose.

Bridge: 5 days of overlap with LMWH and warfarin therapy and an international normalized ratio of 2 or more for 24 hours. Randomized clinical trials show that 5 to 7 days of treatment with unfractionated heparin is as effective as 10 to 14 days of treatment when transitioning to warfarin therapy. If a patient is receiving an adequate warfarin dose, it takes at least 5 days for vitamin K-dependent factor activity levels to decrease sufficiently for therapeutic anticoagulation (INR of 2-3) to occur.

Acetaminophen taken at higher doses (>2 g/day) for >1 week may significantly increase the anticoagulant effects of warfarin. Although the exact mechanism is unclear, this interaction is likely mediated via enzyme inhibition in vitamin K metabolism.

🌿Brussels sprouts and spinach are excellent sources of vitamin K and decrease the anticoagulant effects of warfarin. Ginseng is also known to decrease the serum concentration of warfarin. Their use would ultimately decrease the risk of bleeding episodes and increase thrombosis risk.

Warfarin is the preferred long-term oral anticoagulant in end-stage renal disease patients. It inhibits the synthesis of the vitamin K-dependent clotting factors II, VII, IX, and X and anticoagulant proteins C and S. Warfarin takes several days to become therapeutic and first acts on proteins C and S, causing a transient prothrombotic state; as a result, it generally cannot be started alone. Both low molecular weight heparin (eg, enoxaparin) and rivaroxaban are not recommended in end-stage renal disease (ESRD); they are metabolized by the kidney, so their use in patients with ESRD is associated with increased bleeding risk. Intravenous unfractionated heparin is not contraindicated in ESRD.

How well did you know this?
1
Not at all
2
3
4
5
Perfectly
16
Q

[pulm] Differential Diagnosis of Chronic Dyspnea: Pulmonary Causes

A

COPD

Interstitial lung disease

Pulmonary hypertension

Pleural effusion/hemothorax

Hepatopulmonary syndrome

How well did you know this?
1
Not at all
2
3
4
5
Perfectly
17
Q

COPD

A

COPD is marked by progressive decreases in the expiratory airflow rate, which manifests as a forced expiratory volume in 1 second (FEV1) to forced vital capacity (FVC) ratio of less than 0.7. As airflow limitation increases, more air is trapped during expiration and the residual and total lung volumes increase. Air trapping and airflow obstruction in severe disease also decrease the vital capacity (VC). An accompanying process is destruction of the alveolar-capillary membrane, possibly due to excessive lysis of lung structural proteins.

The alveolar-capillary membrane is destroyed in COPD, resulting in increased lung distensibility and compliance.

Px: Patients are often obese and cyanotic (blue bloater). The mnemonic is BBB = Bronchitis/Blue Bloater.

Dx: CRX reveals increased radiolucency of the lung parenchyma, an elongated and narrow heart shadow, barrel-shaped chest, and a flat diaphragm, which are all the result of air trapping and progressive hyperinflation. Diaphragmatic flattening and muscular shortening caused by hyperinflation result in more difficulty in decreasing intrathoracic pressure during inspiration and therefore increase the work of breathing.

An elevated serum bicarbonate on a chemistry profile may indicate metabolic compensation for a chronic respiratory acidosis; sometimes (but not routinely) ABGs will be necessary to precisely quantify the degree of CO2 retention.

Major Criteria:

Increase in sputum volume

Increase in sputum purulence (generally yellow or green)

Worsening of baseline dyspnea

Additional Criteria:

Upper respiratory infection in the past 5 days

Fever of no apparent cause

Increase in wheezing and cough

Increase in respiration rate or heart rate 20% above baseline

Various nonspecific signs and symptoms may accompany these findings, such as fatigue, insomnia, depression, and confusion

Short-acting β-agonists (SABA) (eg, albuterol, levalbuterol)—also known as “rescue” medications—act within a few minutes of administration, and their effect lasts approximately 4 to 6 hours.

Long-acting β-agonists (LABA) (eg, salmeterol, formoterol, arformoterol) achieve sustained and more predictable improvement in lung function than the short-acting agents.

Vagal stimulation in the lung is mediated via muscarinic receptors. Anticholinergic drugs used to treat COPD include short-acting inhaled agents (eg, ipratropium) and tiotropium, a long-acting inhaled bronchodilator used in stable outpatients. Tiotropium selectively blocks the M3 muscarinic receptor. Short-acting anticholinergic agents are less potent than long-acting β-agonist or long-acting anticholinergic agents.

Tx:

I: Mild; FEV1/FVC <70%; FEV1 ≥80% of predicted (GOLD criteria); With or without chronic symptoms (cough, sputum production)

XOPENEX® (SABA) [Levalbuterol]

DUONEB®: IPATROPIUM {Atrovent®] AND ALBUTEROL (SABA) [Proair®]$

SPIRIVA®: TIOTROPIUM $$$

II: Moderate; FEV1/FVC <70%; FEV1 50% to 80% of predicted; With or without chronic symptoms (cough, sputum production); Add regular treatment with one or more long-acting bronchodilators; add pulmonary rehabilitation.

Mild to moderate exacerbations can be managed at home. Mild exacerbations require treatment with short-acting bronchodilators; moderate exacerbations require short-acting bronchodilators and systemic glucocorticoids and/or antibiotics.

[REDUCE] Short-term (5 days) glucocorticoids is noninferior to conventional 14 day course.

Severe exacerbations are treated in the hospital; severe exacerbations are characterized by loss of alertness or a combination of two or more of the following parameters:

Dyspnea at rest, respiration rate ≥25/min, pulse rate ≥110/min, or use of accessory respiratory muscles.

There is a significant benefit to using antibiotics in patients who have moderate or severe COPD exacerbations. The predominant bacteria recovered are Haemophilus influenzae, Streptococcus pneumoniae, and Moraxella catarrhalis. Generally, antibiotic regimens for community-acquired infection include coverage with a third-generation cephalosporin in combination with a macrolide (azithromycin) or monotherapy with a fluoroquinolone.

5 days

Z PAK

III: Severe; FEV1/FVC <70%; FEV1 30% to 50% of predicted; With or without chronic symptoms (cough, sputum production); Add inhaled corticosteroids if repeated exacerbations

IV: Very severe; FEV1/FVC <70%; FEV1 <30% of predicted or FEV1 <50% of predicted plus chronic respiratory failure; Add long-term oxygen therapy if chronic respiratory failure; consider surgical treatments

🚬 Smoking is the most important risk factor for COPD. In those diagnosed with COPD, smoking cessation has been shown to decrease the rate of decline of FEV1 and decrease mortality.

Long-term supplemental oxygen therapy (LTOT) and lung reduction surgery have shown mortality benefit in specific subpopulations of patients with COPD.

Oxygen therapy is a major component of therapy for very severe (stage IV) COPD and usually is prescribed for patients with arterial PO 2 ≤55 mm Hg or oxygen saturation ≤88% with or without hypercapnia [Resting hypoxemia]. In patients who qualify for continuous therapy because of resting hypoxemia, oxygen treatment should be administered for at least 15 h/day.

Oxygen therapy is the cornerstone of hospital management of COPD exacerbations, with a goal of adequate levels of oxygenation (arterial PO 2 >60 mm Hg or oxygen saturation >90%). Arterial blood gas levels should be measured 30 to 60 minutes after oxygen therapy is started to ensure that oxygenation is adequate without carbon dioxide retention or acidosis.

[NIPPV] Noninvasive intermittent ventilation alleviates respiratory acidosis and decreases respiration rate, severity of dyspnea, and length of hospital stay; importantly, mortality also is reduced.

Indications for noninvasive ventilation include moderate to severe dyspnea with the use of accessory muscles of breathing and paradoxical abdominal motion, moderate to severe acidosis (pH <7.35) and/or hypercapnia (arterial PCO >45 mm Hg), and respiration rate >25/min.

Exclusion criteria include respiratory arrest, cardiovascular instability (hypotension, arrhythmias, myocardial infarction), change in mental status (lack of cooperation), high aspiration risk, viscous or copious secretions, recent facial or gastroesophageal surgery, craniofacial trauma, fixed nasopharyngeal abnormalities, burns, and extreme obesity.

Invasive mechanical ventilation is indicated for patients who cannot tolerate noninvasive ventilation and patients with severe dyspnea with a respiration rate >35/min, life-threatening hypoxia, severe acidosis (pH <7.25) and/or hypercapnia (arterial PCO 2 >60 mm Hg), respiratory arrest, somnolence or impaired mental status, cardiovascular complications (hypotension, shock), or other complications (eg, metabolic abnormalities, sepsis, pneumonia, pulmonary embolism, barotrauma, massive pleural effusion).

Hx: Smoking history, cough, sputum

Px: Diminished breath sounds, wheezing, prolonged expiration, large chest, hyperinflated lungs

Dx: Increased residual volume and increased total lung capacity.

Tx: Low-dose (20 mg) extended-release morphine given daily has been used to relieve dyspnea in patients with advanced COPD.

Cx:

In advanced chronic obstructive pulmonary disease (COPD), destruction of the terminal bronchioles and alveoli causes areas of physiologic dead space to develop. The affected regions have limited surface area available for gas exchange, which leads to ventilation/perfusion (V/Q) mismatch causing local hypoxia and hypercapnia. Hypoxia induces selective vasoconstriction in these areas of the lung and redirects blood flow to better ventilated alveoli, reducing V/Q mismatch.

Supplemental oxygen improves hypoxia but can cause CO2 retention by the following mechanisms:

  • Loss of compensatory vasoconstriction in areas of ineffective gas exchange worsens V/Q mismatch
  • Increase in oxyhemoglobin reduces the uptake of CO2 from the tissues by the Haldane effect
  • Decreased respiratory drive and slowing of the respiratory rate causes reduced minute ventilation

Cx:

The acidosis caused by an acute increase in CO2 increases brain gamma-amino butyric acid (gaba) and glutamine and decreases brain glutamate and aspartate, causing a change in level of consciousness.

Hypercapnia also causes reflex cerebral vasodilation and may induce seizures.

Oxygen should be used cautiously with a goal SaO2 of 90%-93% or PaO2 60-70 mm Hg. Patients who develop significant acidosis or have severely reduced level of consciousness require mechanical ventilation.

How well did you know this?
1
Not at all
2
3
4
5
Perfectly
18
Q

Pulmonary hypertension

A

Defined as mean pulmonary arterial pressure of >25 mm Hg at rest (normal <20 mm Hg).

Pulmonary function in PPH is usually normal, but the elevation in pulmonary artery pressure causes a decrease in cardiac output and eventually right ventricular failure. Patients become dyspneic and hypoxemic due to the mismatch of pulmonary ventilation and perfusion and the reduced cardiac output.

May be idiopathic or related to other disease, such as interstitial lung disease (ILD), chronic thromboembolic disease, other causes (scleroderma, sarcoidosis), or cardiac shunts (atrial septal defect). Primary pulmonary hypertension (PPH) in the United States has been associated with fenfluramines. Primary pulmonary hypertension (PPH) is of unknown etiology and primarily affects women in their thirties or forties.

Group 1: Pulmonary arterial hypertension (PAH), which can be Idiopathic (primary PAH), Heritable, Drug- and toxin-induced, associated with: Connective tissue disease, HIV infection, Portal hypertension, Congenital heart disease, Schistosomiasis (common); long-term responders to calcium channel blockers, w/ overt features of venous/capillaries (PVOD/PCH) involvement, PPH of the newborn syndrome. Tx: Endothelin receptor antagonists (eg, bosentan), phosphodiesterase-5 inhibitors (eg, sildenafil), and/or prostanoids (eg, epoprostenol) are indicated for symptomatic idiopathic PH.

Group 2: Due to: left heart disease, heart failure with preserved LVEF, heart failure with reduced LVEF, Valvular heart disease, Congenital/acquired cardiovascular conditions leading to post-capillary PH. Tx: Management of PH due to LV systolic dysfunction should include 🎡loop diuretics and 🃏 ACE inhibitors (or angiotensin II receptor blockers), often with 🎺beta blockers, and in some cases aldosterone antagonists.

Group 3: Due to: lung disease and/or hypoxia, Obstructive lung disease, Restrictive lung disease, Other lung disease with mixed restrictive/obstructive pattern, Hypoxia without lung disease, Developmental lung disorders. Tx: Oxygen and/or bronchodilator therapy is indicated for PH due to hypoxemia from chronic lung disease.

Group 4: Due to: pulmonary artery obstructions, Chronic thromboembolic PH, Other pulmonary artery obstructions. Tx: Long-term anticoagulation is indicated for patients with PH due to chronic thromboembolic occlusion of pulmonary vasculature.

Group 5: Unclear and/or multifactorial mechanisms : Hematologic disorders, Systemic and metabolic disorders, Others: Complex congenital heart disease,

In general, in the absence of therapy, those with group 1 PAH have worse survival than groups 2 through 5.

Hx: Right ventricular failure develops late in the disease and manifests with right ventricular heave, jugular venous distension, tender hepatomegaly, ascites, edema, etc.

Px: Loud P2, fixed split S2, right-sided S3, pansystolic tricuspid regurgitant murmur, clear lungs or crackles depending on cause.

Dx: CRX shows enlargement of the pulmonary arteries with rapid tapering of the distal vessels (pruning) and enlargement of the right ventricle. An EKG may show right axis deviation, which is secondary to right ventricular strain and hypertrophy due to pulmonary hypertension. Diagnosis can only be confirmed by right heart catheterization or echo.

Untreated pulmonary hypertension would eventually lead to Cor pulmonale.

Tx:

Graded exercise 🏃🏽‍♂️ training improves long-term outcomes (walking distance, peak oxygen consumption, overall functional status) in all patients with PH, with or without LV systolic dysfunction

In refractory cases, heart-lung transplantation (with its considerable risks) may be necessary.

How well did you know this?
1
Not at all
2
3
4
5
Perfectly
19
Q

Pulmonary Edema

A

Occurs when fluid traverses capillary membranes and enters the alveolar space. It is the most common cause of decreased oxygenation in the ICU patient.

Three mechanisms lead to pulmonary edema. These are:

  1. Increased hydrostatic gradient
  2. Diminished oncotic pressure
  3. Increased capillary permeability due to endothelial injury

Any one or more often a combination of these mechanisms will cause fluid to enter the alveolar space.

Dx:

CRX: one or more of the following:

Cephalization of pulmonary vessels, Kerley’s B lines peribronchial cuffing, bat wing pattern, patchy shawdowing with air bronchograms, and increased cardiac size.

Generally, pulmonary edema is bilateral and may change rapidly.

Cardogenic -

Poor cardiac function will cause increased hydrostatic pressures in the pulmonary capillary bed.

Dx: Cardiac edema is usually characterized by:

Interstitial Edema

Interstitial edema occurs as venous pressure rises into the 25-30 mmHg range. Interstitial edema as seen on the chest x-ray may in fact preceed clinical symptoms. Alveolar epithelial junctions are much tighter than endothelial cell junctions. Therefore, excess fluid accumulates in the intersitial space surrounding capillary walls first. Several signs are indicative of interstitial edema. The large pulmonary vessels may begin to lose definition and become hazy. Septal lines may begin to appear.

Kerley’s A lines range from 5 to 10 cm in length and extend from the hila toward the periphery in a straight or slightly curved course. They represent fluid in the deep septa and lymphatics, usually in the upper lobes.

Kerley’s B lines are shorter thin lines (1.5 to 2.0 cm in length) and are seen in the periphery of the lower lung, extending to the pleura. These represent interlobular septal thickening.

The chest x-ray in interstitial edema may take on a diffuse reticular pattern resembling widespread interstitial fibrosis.

Peribronchial cuffing represents interstitial edema and appears as very thick bronchial walls.

Cardiomegaly

Pleural effusions

Peribronchial blurring

Peribronchial cuffing,

“Bat wing” pattern

“Kerley B lines (septal lines)”:

Horizontal lines less than 2 cm long, commonly found in the lower zone periphery. These lines are the thickened, edematous interlobular septa.

Ddx: Pulmonary edema, lymphangitis carcinomatosa and malignant lymphoma, viral and mycoplasmal pneumonia, interstital pulmonary fibrosis, pneumoconiosis, sarcoidosis.

They can be an evanescent sign on the CXR of a patient in and out of heart failure. Represent thickening of interlobular septa.

Remember that Kerley B lines will touch the pleura and blood vessels will not.

“Cephalization”:

In a patient with CHF, the pulmonary capillary wedge pressure rises to the 12-18 mmHg range and the upper zone veins dilate and are equal in size or larger, termed cephalization.

With increasing PCWP, (18-24 mm. Hg.), interstitial edema occurs with the appearance of Kerley lines. Increased PCWP above this level is alveolar edema, often in a classic perihilar bat wing pattern of density. Pleural effusions also often occur.

The initial phase of cardiogenic pulmonary edema is manifested as redistribution of the pulmonary veins. This is know as cephalization because the pulmonary veins of the superior zone dilate due to increased pressure. This diagnosis is made when the upper lobe vessels are equal to or larger in diameter than the lower lobe vessels. The diagnosis of cephalization is more difficult in the supine patient due to gravitational effects.

Alveolar Edema

Alveolar edema occurs when the pulmonary venous pressure exceeds 30 mmHg. Therefore, the signs of interstitial edema are present in patients who have progressed to alveolar edema. Classically, alveolar edema appears as bilateral opacities that extend in a fan shape outward from the hilum in a “bat wing” pattern. As the edema worsens, the opacities become increasingly homogenous. These water-density opacities may contrast with air-filled bronchi which, in normally aerated parenchyma are invisible. The visible appearance of previously imperceptible bronchi is known as air-bronchograms.

Atypical Patterns - Unilateral, miliary and lobar or lower zone edema are considered atypical patterns of cardiac pulmonary edema. A unilateral pattern may be caused by lying preferentially on one side. Unusual patterns of edema may be found in patients with COPD who have predominant upper lobe emphysema.

Pulmonary edema may be unilateral, lobar, miliary, or restricted to the lower zones of the lung. Pulmonary edema may assume any asymmetric or unusual distribution. Although gravitiy as been implicate as the culprit many other theories have been devised to explain the bizarre patterns of pulmonary edema noted. Miliary edema is often considered a normal transitory phase in the development of full scale edema. Lobar or lower zone dema is found in patient suffering from chronic obstructive pulmonary disease with predominate upper lobe emphysema.

One method of differentiating pulmonary edema from other causes of lung opacities is the gravitational shift test. The patient is kept in the supine position for two hours before a chest film is taken. Then the patient is left in the decubitis position with the suspicious hemithorax in the independent position for 2 to 3 hours before a second film is taken. In 85% of patients with pulmonary edema there is a shift in the opacity as opposed to 80% of patient without pulmonary edema who had no shift.

Non-cardiogenic - Can result from volume overload due to renal failure, over hydration, or from diminished oncotic pressure in the liver failure patient, or from endothelial injury as in the patient with ARDS (altered capillary membrane permeability).

“NOT CARDIAC”: Near-drowning, oxygen therapy, transfusion or trauma (fat embolism), CNS disorder, ARDS, aspiration, or altitude sickness, renal disorder or resuscitation, drugs, inhaled toxins, allergic alveolitis, contrast or contusion.

How well did you know this?
1
Not at all
2
3
4
5
Perfectly
20
Q

Congestive Heart Failure

A

The combination of a weak heart and fluid overloading leads to congestive heart failure. Cardiac valvular disease, ischemic cardiomyopathy, renal failure and other causes may also lead to congestive heart failure.

Cardogenic pulmonary edema is the result of left ventricular failure. Initially, increased filling volumes will increase contractility, as described by the Frank-Starling Curve. This mechanism though will fail if the ventricle is overstretched. The result is poor cardiac output and increased pulmonary venous hydrostatic pressures resulting in cardiogenic pulmonary edema.

Dx: The chest radiograph plays an important role in distinguishing fluid overload or congestive failure causes before the onset of symptoms. Left-sided cardiac failure may be detected on a chest x-ray in 25-40% of patients in the event of an acute myocardial ischemia prior to clinical diagnosis. Under ideal situations, the chest film should be taken erect and in the PA view. Supine AP films reduce the viewers ability to detect cardiomegally and redistribution of pulmonary flow. Therefore, semierect and decubitus films are recommended in patients who may have new onset congestive heart failure.

As the left ventricle fails and begins to distend an enlarged cardiac silhouette is seen on x-ray, especially in patients with chronic CHF. This sign, though, is not specific; a pericardial effusion will also enlarge the cardiac silhouette. Also, AP films magnify the cardiac shadow making it difficult to determine actual cardiac enlargment. As pulmonary venous pressures rise pulmonary vessels are recruited in an attempt to normalize pressures. This phenomonan can be seen on chest x-ray as increased pulmonary vascularity with redistrubution to the apex. This signs is also compromised by the typical ICU portable film. Supine position of the patient will cause redistribution of pulmonary flow even in the abscence of CHF. The azygos vein may enlarge as a result of increased pressures transmitted to the venous system. This signs also depends on patient position. The more reliable signs of CHF in the ICU patient are alveolar or intersitial edema. Pleural effusions often accompany subacute or chronic cardiogenic pulmonary edema.

How well did you know this?
1
Not at all
2
3
4
5
Perfectly
21
Q

🏏 Blunt chest trauma (Hemothorax)🥤

A

(eg, sternal bruising) following rapid deceleration (fall >3 m [10 ft] onto pavement)

Because each half of the chest can hold up to 40% of the circulating blood volume, large intrathoracic hemorrhage (eg, hemothorax) can lead to acute hemodynamic instability.

Hemothorax (eg, diminished breath sounds, dullness to percussion) may result from injuries to large (eg, aorta, hilar vessels) or small intrathoracic structures (eg, intercostal blood vessels, lung parenchyma).

Rib fractures (with intercostal vessel injury) are a common cause of hemothorax.

Blunt thoracic 🔴 Aortic injury (BTAI) Rapid deceleration exerts stretching, shearing, and torsional forces capable of rupturing the aorta. The aortic isthmus—the transition zone between the relatively mobile ascending aorta/arch and the fixed descending aorta—is particularly vulnerable to these forces and the most common site of BTAI.

Complete Aortic rupture (ie, tear of the intima, media, and adventitia) typically results in rapid exsanguination and death.

Incomplete rupture (ie, tear of the intima ± media), which may result in:

  • Creation of a secondary, false lumen similar to aortic dissection
  • Creation of an obstructive intimal flap or intramural hematoma that impedes distal blood flow (pseudocoarctation), resulting in proximal hypertension and distal hypotension (eg, upper extremity hypertension with diminished femoral pulses)
  • Expansion of the adventitia under high-flow pressure, causing compression/stretching of surrounding structures such as the left recurrent laryngeal nerve (eg, hoarse voice)

Dx: CT angiography of the chest is highly sensitive and specific for thoracic aortic injury and is readily available. Transesophageal echocardiography (TEE) can also evaluate the thoracic aorta but requires an experienced echocardiographer.

Tx: Hemothorax is treated with tube thoracostomy, which is sufficient to resolve many cases, although if immediate chest tube output is >1,500 mL of blood, emergent surgical thoracotomy is indicated.

Some patients (up to 15%) require emergent thoracotomy (ie, rapidly gaining intrathoracic access - a heroic measure to resuscitate penetrating trauma patients with witnessed or imminent cardiac arrest via open cardiac massage, aortic clamping) for extreme bleeding , including those with:

  • Initial bloody output >1,500 mL
  • Persistent hemorrhage: >200 mL/hr for >2 hours, or continuous need for blood transfusion to maintain hemodynamic stability
How well did you know this?
1
Not at all
2
3
4
5
Perfectly
22
Q

Spontaneous pneumomediastinum

A

Risk factors

  • Asthma exacerbation
  • Respiratory infection
  • Tall, thin, adolescent boy

Clinical features

  • Acute chest pain, shortness of breath, cough
  • Subcutaneous emphysema
  • Hamman sign (crunching sound over heart)

Diagnosis

  • Mediastinal gas on chest x-ray

Treatment

  • Rest, analgesics
  • Avoid Valsalva maneuvers

High intraalveolar pressure due to severe coughing paroxysms can cause air to leak from the chest wall into subcutaneous tissues. Children who cough due to asthma or a respiratory infection are at increased risk for SPM, as are tall, thin adolescent boys.

Presentation often involves acute chest pain and/or shortness of breath. On examination, subcutaneous emphysema is typically palpated in the neck or precordial areas. A crunching sound may be heard over the precordium (Hamman sign). In rare cases, spontaneous pneumothorax (air in the pleural space) can accompany SPM and present with diminished breath sounds on the affected side.

The first step in evaluation is chest x-ray to confirm the presence of mediastinal gas and rule out a Cx: life-threatening pneumothorax that may require emergency needle thoracostomy. An uncomplicated SPM can be managed with rest, pain control, and avoidance of maneuvers that increase pulmonary pressure (eg, Valsalva). Symptoms typically resolve within days to weeks.

💀In the intubated patient the most likely source of air in the mediastinum is pulmonary interstitial air dissecting centripetally. Air in the mediastinum may also originate from tracheobronchial injury or air dissecting through fascial planes from the retroperitoneum. A sudden increase in thoracic pressures (e.g. blunt trauma) may also cause alveolar rupture and consequently pneumomediastinum.

Findings include; streaky lucencies over the mediastinum that extend into the neck, and elevation of the parietal pleura along the mediastinal borders.

Pneumomediastinum often dissects up into the neck. This helps to distinguish it from pneumopericardium that, unlike pneumomediastinum, can extend inferior to the heart.

Causes of pneumomediastinum include; asthma, surgery (post-op complication), traumatic tracheobronchial rupture, abrupt changes in intrathoracic pressure (vomiting, coughing, exercise, parturition), ruptured esophagus, barotrauma, and smoking crack cocaine.

Pneumomediastinum should be distinguished from pneumopericardium and pneumothorax. In pneumopericardium, air can be present underneath the heart, but does not enter the neck.

Continuois diaphram sign

Pneumomediastinum generally will not develop clinical manisfestations. However, a retrosternal crunch is sometimes auscultated (Hamman’s crunch).

Pneumomediastinum rarely causes tension pericardium due to the compressibility of air and the fact that rarely is the pneumomediastinum non-communicating tension due to air is rare. Pneumomediastinum may cause pneumothorax (the reverse is not true) or pneumoperitoneum.

How well did you know this?
1
Not at all
2
3
4
5
Perfectly
23
Q

Hepatopulmonary syndrome

A

Cirrhosis, platypnea (dyspnea sitting up, relieved lying down)

Findings of chronic liver disease, normal pulmonary examination.

Patients with this syndrome often have greater dyspnea (platypnea) and hemoglobin oxygen desaturation in the upright position (orthodeoxia). An upright position increases perfusion to the lower lobes and worsens V/Q matching.

How well did you know this?
1
Not at all
2
3
4
5
Perfectly
24
Q

Differential Diagnosis of Chronic Dyspnea: Other Causes

A

Anemia

Thyrotoxicosis

Neuromuscular disease

Deconditioning

How well did you know this?
1
Not at all
2
3
4
5
Perfectly
25
Q

Anemia

A

History of blood loss or hemolytic disease

Px: Conjunctival pallor

How well did you know this?
1
Not at all
2
3
4
5
Perfectly
26
Q

Thyrotoxicosis

A

Heat intolerance, weight loss, nervousness

Possible goiter

How well did you know this?
1
Not at all
2
3
4
5
Perfectly
27
Q

Neuromuscular disease

A

Dx: Normal cardiac and pulmonary examinations.

Pulmonary function tests show a restrictive pattern without evidence of obstruction and with increased residual volume. Residual volume is increased because of the patient’s inability to exhale fully.

How well did you know this?
1
Not at all
2
3
4
5
Perfectly
28
Q

Deconditioning

A

Situations leading to decreased exercise tolerance

Normal cardiac and pulmonary examinations

How well did you know this?
1
Not at all
2
3
4
5
Perfectly
29
Q

Differential Diagnosis of Acute Dyspnea: Upper Airway Causes:

A

Tracheal stenosis, tracheomalacia

Vocal cord dysfunction

Vocal cord paralysis

Obstructive Sleep Apnea (OSA)

How well did you know this?
1
Not at all
2
3
4
5
Perfectly
30
Q

Tracheal stenosis, tracheomalacia

A

Injury to the trachea from chronic trauma caused by an endotracheal tube may result in inflammation, scarring, and fibrosis or loss of integrity of the tracheal structures, leading to airway narrowing and clinical symptoms. May occur days to months after intubation and is a sequela of the balloon cuff of the tracheal tube pressing against the tracheal wall, causing necrosis and scar tissue formation.

Hx: Prolonged mechanical ventilation and intubation

Px: Stridor, clear lungs, normal cardiac examination

Dx: Best diagnosed on pulmonary function testing, where a characteristic flattening of the curve is observed on flow-volume measurements.

How well did you know this?
1
Not at all
2
3
4
5
Perfectly
31
Q

Vocal cord dysfunction

A

Previous normal spirometry results, history of immediate improvement following intubation

Stridor, clear lungs, normal cardiac examination

How well did you know this?
1
Not at all
2
3
4
5
Perfectly
32
Q

Vocal cord paralysis

A

History of thyroid or neck surgery

Single frequency wheezing localized to throat, dysphonia

How well did you know this?
1
Not at all
2
3
4
5
Perfectly
33
Q

Obstructive Sleep Apnea (OSA)

A

OSA is defined by upper airway narrowing or collapse that results in cessation (apnea) or reduction (hypopnea) in airflow despite ongoing efforts to breathe.

Hx: Loud snoring, gasping, choking?, and pauses in breathing are commonly observed by a bed partner. Subjective symptoms include frequent awakenings, snorting, and nonrestorative sleep. The most important risk factor for OSA is obesity, particularly in patients with prominent distribution of adipose tissue in the trunk and neck. Less important risk factors include male sex, postmenopausal state, family history of OSA, and race. Some possible mechanisms by which obesity can cause OSA include increased upper airway fat deposition, leading to a decrease in airway size and muscle tone as well as reduced lung volume. Central obesity (larger waist-hip ratio) is more important than general obesity. Other risk factors for OSA are larger neck circumference (>17 inches in men; >16 inches in women), nasal narrowing or congestion, large tongue, low-lying soft palate, enlarged tonsils and adenoids, abnormalities of the face or jaw that contribute to airway narrowing, use of muscle relaxants, smoking and alcohol use, and primary medical disorders (acromegaly, androgen therapy, neuromuscular disorders, and stroke).

In patients with OSA, recurrent collapse of the pharynx during sleep results in transient airway obstruction. This causes short periods (20-40 seconds) of hypopnea and apnea, which reduce blood oxygen levels (hypoxia).

The kidneys respond to the hypoxemia by increasing erythropoietin (EPO). EPO stimulates the bone marrow to differentiate more red blood cells (RBCs). Therefore, it is quite common for patients with OSA to have elevated hematocrit levels (polycythemia). Hypertension (due
to hyperadrenergic state), and even cor pulmonale and chronic hypercarbia (due to hypoxia).

The terms polycythemia and erythrocytosis are often used interchangeably. Polycythemia is a laboratory finding of elevated RBC count and hematocrit. Relative polycythemia is generally due to reduced plasma volume, whereas absolute polycythemia (ie, erythrocytosis) is due to increased RBC mass and can be primary (eg, polycythemia vera [PV]) or secondary (eg, due to chronic hypoxia or EPO-producing tumors).

Dx: Polysomnography (PSG) is the gold standard for diagnosis of OSA. Heart rate is monitored. The respiratory pattern is monitored to detect apnea and whether it is central or obstructive. During PSG monitoring, upper airway events are classified as apneas (characterized by complete cessation of airflow) or hypopneas (reductions in airflow), collectively known as disordered breathing events. The apnea-hypopnea index (AHI) is the number of disordered breathing events per hour of sleep and is the standard for measuring the severity of OSA. An AHI of 5 to 15 indicates mild OSA; an AHI of 16 to 30 indicates moderate OSA, and an AHI of more than 30 indicates severe OSA.

Tx: CPAP should be considered first-line therapy in any patient who has OSA and associated symptoms, particularly excessive daytime sleepiness. Optimal positive airway pressure therapy may have salutary effects on cardiovascular diseases that are associated with OSA. Uvulopalatopharyngoplasty, when applied to unselected patients, is effective in less than 50%. 🔪Tracheostomy is a treatment of last resort in severe and refractory sleep apnea;

How well did you know this?
1
Not at all
2
3
4
5
Perfectly
34
Q

Obesity hypoventilation syndrome (OHS)

A

Defined as daytime hypercapnia (PaCO2 >45 mm Hg) in an obese patient (BMI >30 kg/m2, often >40 kg/m2) without another explanation for the hypercapnia. Most patients have coexisting obstructive sleep apnea with frequent apneic events and daytime hypersomnolence. Other features of OHS include dyspnea, polycythemia, respiratory acidosis with compensatory metabolic alkalosis, pulmonary hypertension, and cor pulmonale.

Several mechanisms likely contribute to hypoventilation and resultant hypercapnia. Obesity reduces chest wall and lung compliance, leading to a decrease in tidal volumes and total lung capacity and an increase in airway resistance. As a result, higher levels of ventilatory drive are required to maintain normocapnia, but there is an inability to exhale excess CO2 during the day (due to persistent restriction). This leads to CO2 accumulation overnight, with subsequent chronic respiratory acidosis. Renal bicarbonate excretion is decreased as a compensatory mechanism; this blunts the ventilatory response to the increased CO2 and contributes to hypoventilation.

In sum, patients with OHS “can’t breathe” (due to excess weight and altered lung mechanics) and “won’t breathe” (due to decreased chemosensitivity to hypercapnia from persistent nocturnal hypoventilation).

How well did you know this?
1
Not at all
2
3
4
5
Perfectly
35
Q

Differential Diagnosis of Acute Dyspnea: Psychiatric Causes:

Panic attack /Disorder

A

Sudden panic attacks with acute onset of somatic symptoms that may include chest pain, palpitations, sweating, nausea, dizziness, dyspnea, and numbness.

These symptoms usually last from 5 to 60 minutes. Approximately 50% of patients with panic disorder also have associated agoraphobia, with fears of being in crowds or in places from which escape would be difficult.

Normal cardiac and pulmonary examinations

Tx: Cognitive behavioral therapy (CBT) has been shown to be the most effective psychotherapeutic intervention in controlled trials. Selective serotonin reuptake inhibitors and serotonin-norepinephrine reuptake inhibitors have been shown to be effective.

How well did you know this?
1
Not at all
2
3
4
5
Perfectly
36
Q

Cor pulmonale

A

Right-sided heart failure (RHF) from pulmonary hypertension (PH).

May be idiopathic or due to chronic obstructive pulmonary disease (COPD), interstitial lung disease (eg, idiopathic pulmonary fibrosis), obstructive sleep apnea, pulmonary vascular disease (eg, thromboembolic), or chest wall disorders (eg, kyphoscoliosis).

COPD is the most common cause of cor pulmonale in the United States, with nearly 25% of COPD patients developing this disorder.

Cor pulmonale typically has a gradual onset but can present acutely due to a sudden increase in pulmonary artery pressures (eg, pulmonary embolism). Patients often develop exertional symptoms (eg, dyspnea, angina, syncope).

Physical examination may show loud P2 (pulmonic component of the 2nd heart sound), tricuspid regurgitation murmur (holosystolic at the left lower sternal border), elevated jugular venous pressure (JVP), peripheral edema, hepatomegaly due to hepatic congestion, and possible ascites. COPD patients usually have distant heart sounds due to hyperinflated lungs. End-stage cor pulmonale may present with hypotension, tachycardia, and other signs of cardiogenic shock due to decreased stroke volume.

Chest x-ray may show enlarged central pulmonary arteries and loss of retrosternal air space due to right ventricular hypertrophy. Electrocardiogram usually shows right axis deviation, right bundle branch block, right ventricular hypertrophy, and right atrial enlargement. Right heart catheterization is the gold standard for diagnosis and typically shows elevated central venous pressure, right ventricular end-diastolic pressure, and mean pulmonary artery pressure (>25 mm Hg at rest) without left heart disease.

Tx: Involves optimizing right ventricular dynamics (preload, afterload, and contractility) with supplemental oxygen, diuretics, treatment of underlying etiology, and intravenous inotropes for severe decompensation.

How well did you know this?
1
Not at all
2
3
4
5
Perfectly
37
Q

🌳 Interstitial lung disease (DPLD)

  • Drug-induced
  • Smoking-related
  • Radiation
  • Chronic aspiration
  • Pneumoconioses

DPLD Causing Granulomatous Changes:

  • Hypersensitivity pneumonitis
  • Sarcoidosis
  • Granulomatosis with polyangiitis (Wegener)

Rare DPLD with Well-Defined Features:

  • Lymphangioleiomyomatosis
  • Langerhans cell histiocytosis
  • Anti-GBM disease (Goodpasture syndrome)
  • Chronic eosinophilic pneumonia
  • Pulmonary alveolar proteinosis
  • Cryptogenic organizing pneumonia (COP)

Unknown Causes of DPLD:

  • Idiopathic interstitial pneumonias:
  • Idiopathic pulmonary fibrosis (IPF)
  • Acute interstitial pneumonia

Connective tissue diseases causing DPLD:

  • Rheumatoid Arthritis
  • Systemic Sclerosis
  • Polymyositis/Dermatomyositis
  • Sarcoidosis
  • Granulomatosis aith polyangitis (GPA)
  • Sjogren Syndrome
  • Behçet disease
A

ILD is a collective term referring to multiple etiologies of progressive fibrosis affecting the pulmonary interstitium, alveoli, and conducting airways. Etiologies of ILD include chronic inhalation of organic/inorganic dust (eg, asbestos, beryllium, silicon dioxide), drug toxicity (eg, amiodarone, bleomycin, nitrofurantoin), radiation, and systemic connective tissue disease (eg, rheumatoid arthritis, scleroderma).

Hx: Possible exposure history (silica, asbestos, smoking); collagen vascular disease (scleroderma)

Px: Possible clubbing (pulmonary fibrosis), dry crackles (pulmonary fibrosis)

Dx: In addition, pulmonary function testing (PFT) reveals the characteristic restrictive pattern of decreased FEV1 and FVC and a normal (or sometimes increased) FEV1/FVC ratio. The diffusion capacity of the lung for carbon monoxide (DLCO), which measures gas transfer between alveoli and pulmonary capillary blood, is decreased in ILD (due to fibrosis).

Patients have impaired gas exchange resulting in reduced diffusion capacity of carbon monoxide and increased alveolar-arterial gradient.

How well did you know this?
1
Not at all
2
3
4
5
Perfectly
38
Q

Drug-induced

A

Examples: amiodarone, methotrexate, nitrofurantoin, chemotherapeutic agents (bleomycin)

How well did you know this?
1
Not at all
2
3
4
5
Perfectly
39
Q

“Smoker’s” respiratory bronchiolitis

A

“Smoker’s” respiratory bronchiolitis characterized by gradual onset of persistent cough and dyspnea.

Dx: Radiograph shows ground-glass opacities and thickened interstitium.

Tx: Smoking cessation improves prognosis.

How well did you know this?
1
Not at all
2
3
4
5
Perfectly
40
Q

Radiation

A

May occur 6 weeks to months following radiation therapy

How well did you know this?
1
Not at all
2
3
4
5
Perfectly
41
Q

Pneumoconioses:

A

Berylliosis (aeronautics, electronics)

Bagassosis is a hypersensitivity pneumonitis from exposure to moldy sugarcane.

How well did you know this?
1
Not at all
2
3
4
5
Perfectly
42
Q

🚢 Asbestosis

A

Barbell bodies

A risk for those such as construction workers, shipbuilders, and plumbers who may have long-standing history of exposure to asbestos-containing materials; asbestos mining, shipbuilding, construction, insulation, pipe fitting, plumbing, electrical repair, and railroad engine repair are at risk.

Ninety percent of asbestos-induced pleural abnormalities are caused by pleural plaques (well-circumscribed lesions) and diffuse pleural thickening.

Dx: The diagnosis of asbestosis is based on two essential findings: a convincing history of asbestos exposure with an appropriately long latent period and definite evidence of interstitial 🌳fibrosis at the lung bases.

Bilateral pleural thickening (often calcified, a finding especially evident on CT scan) indicates prior asbestos exposure. Occasionally, the pleural involvement is associated with a ♒ pleural effusion (often with an elevated red cell count) called benign asbestos pleural effusion (BAPE).

Patients have bilateral nodular interstitial pulmonary fibrosis with 🐝 honeycomb changes, as well as characteristic calcified pleural plaques.

Cx: Patients with asbestosis are at risk not only for lung cancer and mesothelioma but also for pharyngeal, gastric, and 🦀 colon cancers.

How well did you know this?
1
Not at all
2
3
4
5
Perfectly
43
Q
A

Rock quary, sand blasting

Caused by the inhalation of crystalline silica.

Occupations typically at risk include cement workers and sandblasters. These workers should be provided with respiratory protection such as a respirator.

Usually, a latency period of 10 to 15 years from first exposure is required for the disease process to become evident. 🥚 Egg shell calcifications, hilar adenopathy.

How well did you know this?
1
Not at all
2
3
4
5
Perfectly
44
Q

Lymphangioleiomyomatosis

A

A rare cystic lung disease that occurs sporadically in women of childbearing age or in association with tuberous sclerosis. Affects women in their 30s and 40s.

Associated with emphysema, spontaneous pneumothorax, or chylothorax in a young woman with dyspnea and a chest radiograph that shows hyperinflation and/or cystic disease should prompt consideration

How well did you know this?
1
Not at all
2
3
4
5
Perfectly
45
Q

Langerhans cell histiocytosis

A

Hx: Affects younger men who 🚬 smoke.

Tx: Improves with smoking cessation.

How well did you know this?
1
Not at all
2
3
4
5
Perfectly
46
Q

Anti-GBM disease (Goodpasture syndrome)

A

Associated with anti-glomerular basement membrane antibody. Hemoptysis and glomerular disease are hallmarks.

How well did you know this?
1
Not at all
2
3
4
5
Perfectly
47
Q

Chronic eosinophilic pneumonia

A

Chest radiograph shows “radiographic negative” of heart failure, with peripheral alveolar infiltrates predominating.

Other findings may include peripheral blood eosinophilia and eosinophilia on bronchoalveolar lavage.

How well did you know this?
1
Not at all
2
3
4
5
Perfectly
48
Q

Pulmonary alveolar proteinosis

A

Slowly progressive disorder affecting patients in their 20s to 50s (predominantly men).

Diagnosed via bronchoalveolar lavage, which shows abundant protein in the airspaces. Chest CT shows “crazy paving” pattern.

How well did you know this?
1
Not at all
2
3
4
5
Perfectly
49
Q

Cryptogenic organizing pneumonia (COP) [BOOP]

A

Patients with COP often present with signs and symptoms consistent with community-acquired pneumonia and may be treated at least once with antibiotics for this presumed diagnosis.

Hx: Most patients have symptoms for less than 3 months, and very few have symptoms for more than 6 months.

Dx: Chest radiograph shows bilateral diffuse alveolar opacities. In the presence of normal lung volume. Alveolar opacities and high-resolution CT shows air-space consolidation, nodular and ground glass opacities, and bronchial wall thickening and dilation, primarily in the lower lung and periphery. One of the key radiographic features of COP is the tendency for opacities to “migrate,” or involve different areas of the lung on serial examinations.

Bronchiolitis obliterans organizing pneumonia (BOOP): COP is the idiopathic form of BOOP.

Many underlying conditions, including certain infectious diseases, collagen vascular diseases, and drug-induced reactions, are associated with the histopathologic features of BOOP and respond best to specific treatment of the primary disease process. Prognosis is typically favorable, with a good response to systemic glucocorticoids.

How well did you know this?
1
Not at all
2
3
4
5
Perfectly
50
Q

Idiopathic pulmonary fibrosis (IPF)

A

Chronic, insidious (>6 mo) (typically follows a prolonged course) onset of a dry, hacking cough and dyspnea, usually in a patient aged >50 y.

Digital clubbing is present in 30% of patients. Risk factors include: history of smoking, organic dust exposure, and age (the prevalence of IPF increases with age). Diagnosis of exclusion.

Dx: Computed tomographic (CT) scan shows the classic findings of IPF, including basal and peripheral disease with septal thickening, evidence of honeycomb changes, traction bronchiectasis, and no evidence of ground-glass opacities or nodules. Although the radiographic findings of IPF are varied, it has a dominant interstitial (reticular) pattern, with or without opacities.

Usual interstitial pneumonia pathology (honeycombing, bibasilar infiltrates with fibrosis).

How well did you know this?
1
Not at all
2
3
4
5
Perfectly
51
Q

Acute interstitial pneumonia

A

Dense bilateral acute (<6 wks) lung injury similar to acute respiratory distress syndrome (ARDS); 50% mortality rate.

How well did you know this?
1
Not at all
2
3
4
5
Perfectly
52
Q

Connective tissue diseases causing DPLD:

A

Tx: Treatment of connective tissue disease associated ILD is unsatisfactory, but cyclophosphamide will slow progression in some patients.

How well did you know this?
1
Not at all
2
3
4
5
Perfectly
53
Q

Rheumatoid arthritis

A

10%-20% of patients with rheumatoid arthritis (mostly men) are affected.

May affect the pleura (pleuritis and pleural effusion), parenchyma, airways (bronchitis, bronchiectasis), and vasculature. The parenchymal disease can range from BOOP-type pattern to usual interstitial pneumonitis.

Tx: Treatment of connective tissue disease associated ILD is unsatisfactory, but cyclophosphamide will slow progression in some patients.

How well did you know this?
1
Not at all
2
3
4
5
Perfectly
54
Q

Systemic sclerosis (diffuse scleroderma)

A

ILD is the most common disease related cause of death in scleroderma.

Dx: High res CT scan will show reticular interstitial thickening and subpleural microblebs. Advanced cases will show thickened fibrotic bands with parenchymal destruction known as honeycombing.

Tx: Treatment of connective tissue disease associated ILD is unsatisfactory, but cyclophosphamide will slow progression in some patients.

Cx:

Nonspecific interstitial pneumonia pathology; may be progressive in 50% of patients. May be exacerbated by aspiration due to esophageal involvement; antibody to Scl-70 or pulmonary hypertension portends a poor prognosis. Monitoring of diffusing capacity for early involvement is warranted.

How well did you know this?
1
Not at all
2
3
4
5
Perfectly
55
Q

Polymyositis/dermatomyositis

A

Systemic sclerosis (diffuse scleroderma)

How well did you know this?
1
Not at all
2
3
4
5
Perfectly
56
Q

🥅 Sarcoidosis

A

Multisystemic disease of unknown cause. The histologic hallmark of the disease is noncaseating granulomas, and staging is based on chest radiograph findings.

Hx: Variable clinical presentation, ranging from asymptomatic to multiorgan involvement. ranging from acute disease with erythema nodosum, fever, arthralgia, and hilar lymphadenopathy (Löfgren syndrome), to a more indolent course. Ninety percent of patients have pulmonary involvement.

Stage 1: hilar adenopathy.

Stage 2: hilar adenopathy plus interstitial lung disease (parenchymal reticular or nodular infiltrates)

Stage 3: interstitial lung disease alone

Stage 4: fibrosis. Noncaseating granulomas are hallmarks.

Dx: Sarcoidosis is a diagnosis of exclusion based on multisystem involvement and histologic evidence of noncaseating granulomas when all other causes have been excluded. Most patients require tissue diagnosis, but some cases do not warrant histologic confirmation. These include classic clinical presentations of known sarcoid syndromes, such as Löfgren syndrome and Heerfordt syndrome (uveitis, parotid gland enlargement, and fever).

The diagnostic method of choice is fiberoptic bronchoscopy with transbronchial biopsy, which will show a mononuclear cell granulomatous inflammatory process. While liver and mediastinal lymph node biopsies are often positive, bronchoscopy is a safer and less invasive procedure.

Tx: Treatment of connective tissue disease associated ILD is unsatisfactory, but cyclophosphamide will slow progression in some patients.

How well did you know this?
1
Not at all
2
3
4
5
Perfectly
57
Q

Granulomatosis with polyangiitis (Wegener)

A

May be associated with upper airway involvement and other systemic findings. + Granulomas

How well did you know this?
1
Not at all
2
3
4
5
Perfectly
58
Q

🥅 Hypersensitivity pneumonitis 🦜

A

Bird Fancier’s lung (antigen mediated)

An allergic, inflammatory lung disease that is also called extrinsic allergic alveolitis. It results from exposure to airborne allergens that cause cell-mediated immunologic sensitization. Immune reaction to an inhaled low-molecular-weight antigen; may be acute, subacute, or chronic.

Hx: Patients typically present with dyspnea, cough, fatigue, anorexia, malaise, and weight loss. Most patients who are exposed to an inhalational antigen have symptoms within 4 to 12 hours.

Dx: Pulmonary function testing may show obstructive and restrictive defects. Noncaseating granulomas are seen.

Chronic hypersensitivity pneumonitis has a poor prognosis.

Nitrofurantoin can cause an acute hypersensitivity pneumonitis. This condition can progress to a chronic alveolitis with pulmonary fibrosis. The presenting symptoms are fever, chills, cough, and bronchospasm. In addition, the patient may experience arthralgias, myalgias, and an erythematous rash. The chest x-ray will show interstitial or alveolar infiltrates. CBC often shows leukocytosis with a high percentage of eosinophils. Tx: The treatment is to discontinue the nitrofurantoin, and to begin corticosteroids.

How well did you know this?
1
Not at all
2
3
4
5
Perfectly
59
Q

💀💀💀CRX

A

Index

How well did you know this?
1
Not at all
2
3
4
5
Perfectly
60
Q

Firsts

A

Turn off stray lights, optimize room lighting, view images in order

Patient data:

Name

Date

PA/AP

Uprigt/supine

61
Q

Quality

A

Rotated?

Penetration? (Thoracic spine seen through heart)

All areas included (costophrenic angles)

Inflation (3 cm diaphragmatic curvature; 8 -10 posterior ribs in nl)

62
Q

Airway

A

Deviation of the trachea (related to a mediastinal hematoma)

Masses in the airway

Airway compression

63
Q

Bone

A

Lesions or fractures

Clavicles

Soft tissue calcification

RUG (gallstones, free air)

64
Q

CardioMediastinum

A

Cardiothoracic ratio

RA, LV, LA

Mediastinal contour: width? mass?

Lines (R paratracheal, Azygous, SVC, Aorta, Azygoesophageal line, descending aortic line)

Widened mediastinum:

Loss of the normal clear aortic arch contour “knob”

Loss of the appearance of a normal descending thoracic aorta (no ‘lateral aortic silhouette’ is seen).

Deviation of nasogastric tubes to the right (Indicating a mediastinal hematoma pushing the esophagus to the right side).

Left apical pleural ‘capping’

There is is a rind of fluid above the left lung apex where blood has tracked posteriorly over the left apex.

Signs of a supine pleural effusion

5 T’s Thymoma, teratoma, thyroid, traumatic aorta, parathyroid mass

65
Q

Diaphragm

A

Sharp border

Costophrenic angles sharp bilaterally

Air under diaphragm

66
Q

Effisions (Pleura, Pericardial)

A

Lucencies (pneumothorax)

Thickeing, nodularity, calcification, or effusions

67
Q

Fields (Lung)

A

Lung zones symmetrical?

Parenchyma (focal or diffuse abnormalitis)

Interstitial and vascular markings (size, prominence)

Lucency (pneumothorax), cavity, or abnormal shadowing (companion shadow of the second rib)

Hila (l higer than right; branching pattern)

68
Q

Solitary Pulmonary Nodule

A

Factors increasing malignant probability

  • Large size (>2 cm independently correlates with >50% malignant probability).
  • Advanced patient age
  • Female sex
  • Active or previous smoking
  • Family or personal history of lung cancer
  • Upper lobe location
  • Spiculated radiographic appearance

A differential of possible etiologies is as follows:

Granuloma – usually caused by fungal infections like histoplasmosis or tuberculosis

Lung Carcinoma

Solitary metastasis – usually from colon, breast, kidney, ovary, or testis

Round pneumonia

Abscess

Round atelectasis

Hamartoma – popcorn calcification is sometimes seen

Sequestration

Arteriovenous malformation

Other things can cause an apparent nodule but are actually outside the lung including:

Fluid in an interlobar fissure

Pleural plaques – small, often calcified, plate-like surfaces on the pleura often caused by asbestos fibers that invade the pleura from the lungs

Skin lesions – nipple shadow, mole, lipoma, etc.

Low Risk Patient

≤ 4mmNo follow-up needed

4-6mm12 mo; if no change - stop

6-8mm6-12 mo; no change - follow-up at 18-24 mo

> 8mmCT follow-up at 3, 9, 24mo or PET/CT, or biopsy

High Risk Patient (eg. smoking history or history of malignancy)

≤ 4mm 12 mo; if no change - stop

4-6mm 6-12mo; no change - follow-up at 18-24 mo

6-8mm 3-6mo; no change - follow-up at 18-24 mo

> 8 mm CT; follow-up at 3, 9, 24mo or PET/CT, or biopsy or surgical excision

An initial CT scan will also help evaluate whether the lesion can be accessed percutaneously without risk for pneumothorax.

Percutaneous biopsy is recommended for lesions that appear malignant and in patients with high clinical suspicion for malignancy. If the CT findings are suspicious for malignancy, a biopsy can be attempted by CT guidance.

69
Q

Post-primary TB:

A

Focal patchy airspace disease “cotton wool” shadows, cavitation, fibrosis, nodal calcification, and flecks of caseous material. These occur most commonly in the posterior segments of the upper lobes, and superior segments of the lower lobes.

70
Q

Pulmonary hemorrhage:

A

Blood fills the bronchi and eventually the alveoli.

Has an appearance like that of other airspace filling processes (pneumonia, edema) which have opacity often with air bronchograms.

Caused by trauma, Goodpastrue’s syndrome, bleeding disorders, high altitude, and mitral stenosis.

Notable in that it may clear more quickly than other alveolar densities such as pneumonia.

71
Q

Interstitial pulmonary fibrosis

A

The six most common causes of diffuse interstitial pulmonary fibrosis are idiopathic (IPF, >50% of cases), collagen vascular disease, cytotoxic agents and nitrofurantoin, pneumoconioses, radiation, and sarcoidosis. Clinically the patient with IPF will present with progressive exertional dyspnea and a nonproductive cough. Radiographically, IPF is associated with hazy “ground glass” opacification early and volume loss with linear opacities bilaterally, and honeycomb lung in the late stages. IPF carries a poor prognosis with death due to pulmonary failure usually occurring within 3-6 years of the diagnosis unless lung transplant is performed.

72
Q

🛍 Emphysema

A

Loss of elastic recoil of the lung with destruction of pulmonary capillary bed. This causes collapse of the small airways and prolongs the expiratory phase of respiration.

Px: During the prolonged expiration, patients will have rapid and shallow breathing and purse their lips to avoid collapse of the small airways (this causes auto–positive end-expiratory pressure [auto-PEEP]).

Commonly seen on CXR as diffuse hyperinflation with flattening of diaphragms, increased retrosternal space, bullae (lucent, air-containing spaces that have no vessels that are not perfused) and enlargement of PA/RV (secondary to chronic hypoxia) an entity also known as cor pulmonale. Hyperinflation and bullae are the best radiographic predictors of emphysema. However, the radiographic findings correlate poorly with the patientâs pulmonary function tests. CT and HRCT (high resolution CT) has emerged as a technique to evaluate different types, panlobular, intralobular, paraseptal and for guidance prior to volume reduction surgery.

Occasionally the trachea is very narrow in the mediolateral plane in emphysema. “Saber sheath” tracheal deformity is when the coronal diameter is less than 2/3 that of the sagittal.

In smokers with known emphysema the upper lung zones are commonly more involved than the lower lobes. This situation is reversed in patients with alpha-1 anti-trypsin deficiency, where the lower lobes are affected.

Chronic bronchitis commonly occurs in patients with emphysema and is associated with bronchial wall thickening.

alveolar septa.

“loculated”

Cx: Patients with emphysema or chronic bronchitis at baseline often have a thin body habitus due to caloric expenditure in excess of intake, increased anteroposterior thickness of the thorax (barrel chest), clubbing, productive cough, and often employ pursed-lip breathing to prolong the expiratory phase of respiration and prevent sudden collapse of the small airways. When these patients have an exacerbation severe enough to warrant possible
intubation, they often use accessory muscles of respiration, assume a “tripod” position to facilitate the movement of these muscles and may display nasal flaring and cyanosis as well.

73
Q

Pneumopericardium

A

Pneumopericardium, an uncommon occurence, is most often found in the post-operative cardiac patient. Radiographically, pneumopericardium appears as a lucent area around the heart extending up to the main pulmonary arteries.

A lucent stripe along the inferior border of the cardiac silhouette which crosses the midline is also diagnostic for pneumopericardium “continuous diaphragm sign”

74
Q

Diaphragmatic hernia

A

There are 3 types of diaphragmatic hernia that may be seen in CXR.

By far the most common is a hiatal hernia - the stomach slips through the esophageal hiatus into the chest.

A Bochdalek hernia is through a weakness in the diaphragm, and usually occurs on the left side posteriorly (Bochdalek - back and to the left).

Morgagni hernias typically occur medially. Weakness of the diaphragm can occur without frank herniation of abdominal contents. This is termed an eventration, and it usually occurs on the right with a portion of the liver bulging cephalad.

75
Q

🌆 Community Acquired Pneumonia (CAP)

A

IDSA/ATS Minor Criteria for Severe Community-Acquired Pneumonia

Clinical Criteria

  • Confusion (new-onset disorientation to person, place, or time)
  • Hypothermia (core temperature <36.0°C [96.8°F])
  • Respiration rate ≥30 breaths/mina
  • Hypotension necessitating aggressive fluid resuscitation
  • Multilobar pulmonary infiltrates

Laboratory Criteria

  • Arterial PO 2/FIO 2 ratio ≤250a
  • Leukopenia (<4000 cells/µL [4.0 × 109/L])
  • Thrombocytopenia (<100,000 platelets /µL [10 × 109/L])
  • Blood urea nitrogen >20 mg/dL (7.1 mmol/L)

CURB-65 Identifies ❗high-risk patients and to predict a complicated course. Patients who meet at least two criteria are usually admitted to the hospital (mortality rate 8.3%), and those with at least three criteria are considered for intensive care unit (ICU) admission (mortality rate 20%).

Confusion, blood Urea nitrogen >20 mg/dL, Respiration rate ≥30 breaths/min, systolic Blood pressure <90 mm Hg OR diastolic blood pressure <60 mm Hg, and age ≥65 years)

Px: Dullness to percussion and increased tactile fremitus (vibrations that are perceived on palpation). On auscultation, instead of normal vesicular breath sounds, patients often have bronchial breath sounds (like breathing through a straw, normally heard only over the trachea) and inspiratory crackles, as well as other findings like bronchophony (sounds like “99” are louder and clearer), egophony (long “e” perceived as short “a,” like a goat bleating), and whispered pectoriloquy (whispered sounds are louder and clearer).

Tx (Outpatient): 🐦Macrolide (azithromycin, clarithromycin, or erythromycin) or doxycycline

Risk factor(s) for drug-resistant S. pneumoniae or underlying comorbidities

Respiratory fluoroquinolone (moxifloxacin, gemifloxacin, or levofloxacin) or β-lactam plus a macrolide or doxycycline

Tx (Inpatient):

Medical ward

β-lactam plus a macrolide or doxycycline; or respiratory fluoroquinolone (eg, moxifloxacin, gemifloxacin or levofloxacin)

Intensive care unit

β-lactam plus either azithromycin or a fluoroquinolone; if penicillin allergic, a respiratory fluoroquinolone plus aztreonam

🏇🏽Streptococcus pneumoniae

Other pathogens responsible for CAP include M. pneumoniae, viruses, and C. pneumoniae.

Modifying Factors That Increase the Risk of Infection With Specific Pathogens

Age >65 y, β-lactam therapy (in previous 3 mo), alcoholism, immunosuppression (illness, glucocorticoids), multiple medical comorbidities, exposure to a child in a day care center

Penicillin-resistant and drug-resistant pneumococci

Residence in an extended care facility (HAP), underlying cardiopulmonary disease, multiple medical comorbidities, recent antibiotic therapy

🔮 Enteric gram-negative bacteria The offending organisms often include Pseudomonas species, E-coli, Klebsiella species, and Proteus species

Pseudomonas aeruginosa 🛀🏿

Structural lung disease (bronchiectasis), glucocorticoids therapy, broad-spectrum antibiotic therapy for >7 d; malnutrition

Dx: Pseudomonas aeruginosa or 🏮gram-negative rods on sputum Gram stain

Tx: Antipseudomonal β-lactam with pneumococcal coverage (eg, cefepime, imipenem, meropenem, or piperacillin-tazobactam) ➕ ciprofloxacin or levofloxacin (750 mg) OR antipseudomonal β-lactam with pneumococcal coverage plus an aminoglycoside plus azithromycin; or antipseudomonale β-lactam with pneumococcal coverage plus an aminoglycoside plus a respiratory fluoroquinolone

Endobronchial obstruction (tumor)

Anaerobes, Streptococcus pneumoniae, Haemophilus influenzae, Staphylococcus aureus

💉Intravenous drug use

Anaerobes, S. aureus, Mycobacterium tuberculosis, S. pneumoniae

Influenza epidemic in the community

Influenza virus, S. pneumoniae, S. aureus, H. influenzae

COPD, 🚬 smoking history

S. pneumoniae, 🍒Haemophylis influenzae, Moraxella catarrhalis, P. aeruginosa, Legionella species, Chlamydophila

Poor 🦷dental hygiene, aspiration, lung abscess

Oral anaerobes

Animal exposure

Coxiella burnetii (farm animals); Chlamydophila psittaci, Cryptococcus (birds); Histoplasma (birds, bats)

PR/MDR 💪🏿Pneumococcal Pneumonia

Hx: Age >65 y, β-lactam therapy (in previous 3 mo), alcoholism, immunosuppression (illness, glucocorticoids), multiple medical comorbidities, exposure to a child in a day care center.

Dx: CA-MRSA or compatible sputum Gram stain

Tx: Add 🚐vancomycin OR 🚧 linezolid to β-lactamb ➕ either 🐦azithromycin or a fluoroquinolone.

Patients with bacteremic pneumococcal pneumonia should be discharged on oral amoxicillin to complete 7 days of therapy.

76
Q

HAP (hospital acquired or nosocomial pneumonia)

A

HAP (or nosocomial pneumonia) is pneumonia that occurs 48 hours or more after admission and did not appear to be incubating at the time of admission.

The most likely organisms are P. aeruginosa, Staphylococcus aureus (often methicillin resistant), Enterobacter, Klebsiella pneumoniae, and Escherichia coli.

🔮 Enteric gram-negative bacteria The offending organisms often include Pseudomonas species, E-coli, Klebsiella species, and Proteus species

77
Q

Organizing pneumonia

A

Subacute illness (4-6 wk) with fever and alveolar infiltrates (often peripheral). Diagnosis is based on biopsy (transbronchial or open lung) or characteristic clinical picture and response to glucocorticoids.

78
Q

Eosinophilic pneumonia

A

Presents as an acute illness with the radiographic “photo negative” of pulmonary edema, usually with peripheral eosinophilia. Biopsy may not be needed with a classic presentation.

79
Q

Hypersensitivity pneumonitis

A

Recurrent episodes of fever and dyspnea, with rapid resolution of infiltrates; chronic infiltrates after multiple episodes. Diagnose with precipitating antibodies to the antigen (molds etc.), characteristic history, or open lung biopsy.

80
Q

Legionella Pneumonia

A

Hx: Suspect in patients with risk factors (eg, age ≥50 years, smoking history, immunocompromising condition) who present with severe pneumonia and extrapulmonary symptoms (eg, headache, confusion, diarrhea, kidney failure).

Ubiquitous high fever above 40 degrees celsius, bradycardia relative to the fever, abdominal complaints (diarrhea in up to half of cases and nausea and vomiting as well), scanty cough that is occasionally blood-streaked, and laboratory abnormalities such as hyponatremia, elevated liver function tests and elevated creatine phosphokinase.

Pontiac fever is an acute, self-limited, flu-like illness due to Legionella, but it does not cause pneumonia.

Dx: Hyponatremia (< 136mEq/L); The urine antigen test detects only Legionella serogroup 1.

81
Q

Pneumocystis pneumonia

A

Hx: Subacute onset of dyspnea on exertion, hypoxia, nonproductive cough; diffuse interstitial infiltrates on chest radiograph

Dx: Chest x-ray usually shows bilateral interstitial infiltrates. Hypoxia out of proportion to the radiographic findings is also suggestive. Serum lactate dehydrogenase levels are frequently elevated. The diagnosis is confirmed by demonstration of the organism in sputum or bronchoalveolar lavage aspirate.

Tx: Trimethoprim-sulfamethoxazole (TMP-SMX) is the initial drug of choice for the treatment of PCP regardless of pneumonia severity. Treatment typically lasts for 21 days. Adjunctive corticosteroids have been shown to decrease mortality in cases of severe PCP (possibly by reducing inflammation due to dying organisms). Indications for corticosteroid use include partial pressure of oxygen (PaO2) <70 mm Hg or an alveolar-arterial (A-a) gradient >35 mm Hg on room air. This patient has a PaO2 of 54 mm Hg on room air, indicating the need for corticosteroid use.

82
Q

Lung abscess

A

Periodontal disease and a history of aspiration, often as a result of loss of consciousness due to seizure, alcoholism, or illicit drug use, predispose to this anaerobic infection. Patients complain of several days or weeks of malaise and fever and eventually develop chills, cough, pleuritic chest pain, and cough productive of putrid sputum.

Due to position at the time of loss of consciousness and to the anatomy of the lung, the lung segments most often involved in lung abscesses include the posterior segment of the right upper lobe (wide, short, and vertically placed) and the superior segments of both lower lobes.

May be acute or indolent; causes indolent symptoms of fever, cough, dyspnea, weight loss and CT scan findings of an infiltrate with a cavity. Cavities greater than 2 cm are described as lung abscesses.

  • *Hx:** Patients usually have foul-smelling sputum.
  • *Dx:** Usually have necrotic tissue with air fluid levels
83
Q

Histoplasmosis or coccidioidomycosis

A

Histoplasmosis is an endemic mycosis of the central and midwestern United States that rarely causes illness in immunocompetent individuals but may occasionally cause subacute pulmonary symptoms (<5%). However, pulmonary histoplasmosis usually causes focal infiltrates with hilar lymphadenopathy

Hx: Patients may have fever, cough, and night sweats. These diseases are usually geographically specific (eg, histoplasmosis, Midwest; coccidioidomycosis, Southwest).

Dx: Chest radiography may show a miliary (eg, histoplasmosis) or cavitary lesion.

Cx: Mediastinitis

84
Q

Granulomatosis with polyangiitis (Wegener granulomatosis)

A

Necrotizing granulomas in the lung and necrotizing glomerulonephritis. Patients have fever and cough. Chest radiography shows a cavitary lesion in up to 70% of cases.

85
Q

Actinomycosis

A

Characterized by cough, hemoptysis, and (eventually) draining sinuses. Has an indolent course; patients may have respiratory symptoms for up to 5 mo before diagnosis. Sulphur granules are seen in stained specimens from draining sinuses.

86
Q

Tubercuosis (TB)

A

Hx: Weight loss, cough.

Dx:

CBC: Anemia is present in 10% of patients with TB, particularly if infection is disseminated. Leukocytosis is present in 10% of patients with TB.

BMP: Hyponatremia is present in up to 11% of patients with TB.

CRX: The classic appearance of reactivation TB is lesions in the apical posterior segments of the upper lung and superior segments of the lower lobe. Chest radiograph results may be normal in patients with endobronchial disease or in symptomatic HIV-infected patients with active TB. In disseminated disease, 50%-90% have a miliary pattern on chest x-ray.

TST: A positive test result may indicate latent TB. False-positive results can occur with exposure to nontuberculous mycobacteria and BCG vaccine. False-negative results occur in anergic patients and in up to 25% of patients with active TB.

≥5 mm: HIV patients or HIV at-risk patients; close contacts of patients with active TB; persons with CXR showing healed TB

≥10 mm: Immigrants; intravenous drug abusers; patients with underlying medical conditions increasing the risk for TB like diabetes and chronic kidney disease;
residents and employees of health-care facilities, nursing homes, prisons, and mental institutions

≥15 mm: All other persons

IGRA: IGRA is more specific than, and at least as sensitive as TST. It is preferred to TST in the setting of previous BCG vaccination and in individuals who are unlikely to return to have the test interpreted. Like TST, IGRA cannot distinguish between LTBI and active infection.

Sputum smear for acid-fast bacilli: At least 5000 to 10,000 organisms should be present for a smear to be positive. The more acid-fast bacilli seen, the more infectious is the patient. Induced sputum or gastric washings may be obtained if a patient does not have a productive cough. Nontuberculous mycobacteria may produce positive smears. Nocardia is acid fast on the modified acid-fast stain.

Sputum culture for acid-fast bacilli: Solid media cultures in conjunction with liquid media are often the gold standard used for diagnosis. The only false-positive results that occur are as a result of laboratory error or contamination of the specimen. False-negative results do occur and are often due to nontuberculous mycobacterial overgrowth and antibiotic treatment.

Nucleic acid amplification of smear-positive sputum: Results are available in a few hours. False-positive results occur only with laboratory contamination, although the test does not indicate if bacteria are alive or dead (ie, may remain positive for some time after treatment).

Tx:

The standard treatment for suspected or confirmed active tuberculosis is at least 6 months of a four-drug regimen usually consisting of isoniazid, rifampin, pyrazinamide, and ethambutol.

Patients with LTBI are treated with one of the following treatment regimens:

Isoniazid daily for 9 months OR

Rifampin daily for 4 months OR

A combination of rifapentine and isoniazid once weekly for 3 months via directly observed therapy.

Health-care workers with a positive (≥ 10 mm) PPD require chemoprophylaxis, optimally with isoniazid for 9 months. Alternative regimens include a combination of rifampin and pyrazinamide or, if the other drugs are not well-tolerated, rifampin alone.

87
Q

Aspergillus

A

A ubiquitous fungus that most people encounter daily. Conidia are inhaled into the lung and convert to potentially pathogenic hyphae. Patients with immunocompetency rapidly clear the organism and rarely develop infection; however, a subset of immunocompetent patients with a history of pulmonary disease (eg, cavitary tuberculosis) may develop chronic pulmonary aspergillosis (CPA) at sites of lung damage. Diagnosis is made by the presence of all 3 of the following:

>3 months of symptoms - fever, weight loss, fatigue, cough, hemoptysis, and/or dyspnea

Cavitary lesion(s) containing debris, fluid, or an aspergilloma (fungus ball)

Positive Aspergillus IgG serology

Tx: Therapy depends on symptoms and severity of disease; antifungal medication (eg, itraconazole, voriconazole), surgery (to prevent hemoptysis), and bronchial artery embolization (for hemoptysis with extensive disease) may be used together or separately.

88
Q

Solitary Pulmonary Nodule

A

A solitary pulmonary nodule (SPN) is defined by the following features:

  • Rounded opacity
  • ≤3 cm in diameter (>3 cm is considered a “mass”)
  • Surrounded by pulmonary parenchyma
  • No associated lymph node enlargement

In addition to size, other factors that influence the probability of an SPN being malignant include patient age, sex, smoking history, family history, location of the nodule in the lung, and radiographic appearance of the nodule (eg, regular versus irregular borders). Nodules >0.8 cm that are intermediate or high probability for malignancy (ie, ≥5% risk) based on these factors require tissue diagnosis with biopsy or surgical excision. This relatively large SPN with irregular borders in a 65-year-old patient with a significant smoking history has high malignant probability and should be biopsied or surgically excised.

The size of an SPN strongly correlates with the chances of it being malignant. Nodules <0.6 cm are unlikely to be malignant and generally do not require follow-up; however, nodules >0.8 cm require additional management or surveillance.

Low Risk Patient

  • ≤ 4mm: No follow-up needed
  • 4-6mm: 12 mo; if no change - stop
  • 6-8mm: 6-12 mo; no change - follow-up at 18-24 mo
  • > 8mm: CT follow-up at 3, 9, 24 mo OR PET/CT, OR biopsy

High Risk Patient (eg. 🚬smoking history or history of malignancy)

  • ≤ 4mm 12 mo; if no change - stop
  • 4-6mm 6-12 mo; no change - follow-up at 18-24 mo
  • 6-8mm 3-6 mo; no change - follow-up at 18-24 mo
  • > 8 mm CT; follow-up at 3, 9, 24 mo or PET/CT, or biopsy

An initial CT scan will also help evaluate whether the lesion can be accessed percutaneously without risk for pneumothorax.

Percutaneous biopsy is recommended for lesions that appear malignant and in patients with high clinical suspicion for malignancy. If the CT findings are suspicious for malignancy, a biopsy can be attempted by CT guidance.

A differential of possible etiologies is as follows:

  • Granuloma – usually caused by fungal infections like histoplasmosis or tuberculosis
  • Lung Carcinoma
  • Solitary metastasis – usually from colon, breast, kidney, ovary, or testis
  • Round pneumonia
  • Abscess
  • Round atelectasis
  • Hamartoma – popcorn calcification is sometimes seen
  • Sequestration
  • Arteriovenous malformation

Other things can cause an apparent nodule but are actually outside the lung including:

  • Fluid in an interlobar fissure
  • Pleural plaques – small, often calcified, plate-like surfaces on the pleura often caused by asbestos fibers that invade the pleura from the lungs
  • Skin lesions – nipple shadow, mole, lipoma, etc.
89
Q

Lung Cancer

A

About 10% of lung cancers present with a paraneoplastic syndrome.

Small cell carcinoma

Tumors producing ADH or ACTH are overwhelmingly SCLCs, which arise from
hormonally active neuroendocrine cells. SCLC is a rapidly growing neoplasm; early mediastinal involvement,

Adenocarcinoma

Adenocarcinoma is usually a peripheral lesion that is usually not associated with hypercalcemia. It is typically associated with hypertrophic pulmonary osteoarthropathy.

Squamous Cell Carcinoma

Increased thirst, and easy fatigability) and laboratory studies are consistent with hypercalcemia, which is usually associated with the above mentioned carcinoma (remember: “sCa++mous”).

Hypercalcemia usually result from the effects of parathyroid hormone-related protein (PTHrP), which is similar in nature to PTH in the receptor-binding area. Binding to PTH receptor results in increased calcium resorption from the bones and increased renal calcium resorption in the distal tubule. Furthermore, hypercalcemia in such settings may result from metastatic involvement of the bone and usually develops as a late complication of the cancer; thus, its appearance has very serious implications.

Cx:

SVC syndrome: Tumors that may cause SVC syndrome include small-cell carcinoma of the lung, squamous cell carcinoma of the lung, lymphoma, and mediastinal tumors like thymomas and germ cell tumors.

Superior pulmonary sulcus (SPS) tumor (Pancoast tumor) Usually a malignant lung neoplasm (most commonly squamous cell carcinoma or adenocarcinoma). A complication when it extends into the apex. Patients have compression of the C8, T1, and T2 nerves and often complain of arm and shoulder pain.

Tumors located in the SPS often present with shoulder pain as the initial symptom due to invasion of the brachial plexus or adjacent structures. The pain may also radiate up to the head and neck or down the ipsilateral arm in the ulnar nerve distribution, and weakness and atrophy of the medial hand muscles may occur. Horner syndrome is also common and occurs due to tumor invasion of the paravertebral sympathetic chain and inferior cervical ganglion. Initial evaluation includes chest imaging (eg, x-ray of the chest) to evaluate for lung mass.

Adenocarcinoma – (35-50%) Peripheral, sometimes associated with scars, high incidence of early metastasis

Squamous Cell Carcinoma – (30%) Central, with hilar involvement, cavitation is common, slow growing

Small Cell - (15-20%) Central, cavitation is rare, hilar and mediastinal masses often the dominant feature, rapid growth and early metastases. If disease is confined to the chest (i.e., limited stage) as in this patient, then chemotherapy is initiated with the addition of radiation to the chest concurrent with the first or second cycle of chemotherapy. In this setting, radiation decreases rates of a local recurrence and increases median survival. Routine use of chest radiotherapy in extensive-stage disease does not prolong survival.

Large cell – (10-15%) Peripheral, large, cavitation present

Bronchaveolar – (3%) Peripheral, rounded appearance, pneumonia-like infiltrate (air bronchograms), occasionally multifocal

Carcinoid and bronchial gland tumors (less than 1%) are called bronchial adenomas but are actually low-grade malignant neoplasms. They are resistant to radiation and chemotherapy. Patients present usually before the age of 60 years; common symptoms include hemoptysis, chronic cough, focal wheezing, and recurrent pneumonia (due to obstruction and atelectasis). The chest radiograph may be normal. The classic presentation of flushing, diarrhea, wheezing, and hypotension (carcinoid syndrome) is rare. Typically a well defined endobronchial lesion; nodal, liver and brain metastases may enhance densely (i.e. They may be hypervascular).

Dx: These tumors are centrally located, and fiberoptic bronchoscopy will offer tissue diagnosis of a tumor in a central airway. CT scanning and octreotide scintigraphy also may help to localize the lesion.

90
Q

Hilar Adenopathy

A

A differential of possible etiologies can be broken up into three different categories:

Inflammation (sarcoidosis, silicosis)

Neoplasm (lymphoma, metastases, bronchogenic carcinoma)

Infection (tuberculosis, histoplasmosis, infectious mononucleosis)

An important consideration to keep in mind is that since the pulmonary arteries also course through the same area, enlargement of these vessels may be confused with hilar adenopathy. Typically, lymphadenopathy has a more lumpy-bumpy appearance, while an enlarged pulmonary artery appears smooth.

91
Q

💀💀💀 Tubes, Lines, Drains

A

Dobhoff - weighted end; specific for feeding

Central line

Peripheral IV

PICC Line

IJ Catheter

Pacemaker - SC tissue -

LVAD -

Pulmonary Artery Catheter -

Intra-aortic baloon pump -

Left ventricular assist device -

92
Q

Endotracheal tubes (ET Tubes) or tracheostomy tubes

A

Cuffed conduits placed in the trachea either through the oropharynx or through a surgically created tracheostomy. These tubes maintain airway patency and allow for mechanical ventilation of patients with respiratory failure.

A tracheostomy is generally performed in patients who are intubated for longer than 1-3 weeks or who have upper airway obstruction.

The carina can be assumed to be at the T4-T5 interspace, given that 95% of patients’ carinas project over the T5, T6, or T7 vertebral bodies.

The Dee method for approximating the position of the carina involves defining the aortic arch and then drawing a line inferomedially through the middle of the arch at a 45 degree angle to the midline. The intersection of the midline and the diagonal line is the most likely position of the carina.

Approximately, 10% of endotracheal tubes are malpositioned. The tube is more likely to enter the right main stem bronchus, due to its more vertical orientation, and reduce left lung ventilation, leading to collapse of the left lung. If the endotracheal tube enters into the bronchus intermedius, the right upper lobe can also collapse. Superiorly placed ET tubes may enter the pharynx or dislodge from the trachea into the esophagus causing filling of the stomach with air and, potentially, reflux of gastric contents. The glottis may also be damaged.

Major complications from endotracheal tubes are unusual. These include tracheal stenosis, tracheal ruputure, cord paralysis, cervical mediastinal emphysema, hematoma, and abscess formation.

93
Q

Nasogastric and Feeding Tubes

A

Inserted through the nares and into the stomach. They are used for gastric decompression or feeding. Generally a chest x-ray is not necessary following the placement of a nasogastric tube.

Feeding tubes are generally placed into the proximal small bowel, as confirmed by an abdominal film. A chest x-ray may be obtained following the insertion of small-bore feeding tubes to rule out placement within the lung, which may have serious consequences. Also, patients who are status-post esophagectomy should receive a chest x-ray to evaluate the placement of any nasogastric tube.

94
Q

Central (line)[IJ] Venous Pressure Monitors

A

The intravascular volume status of critically ill patients is crucial to their management.

A central venous pressure can be obtained directly via central vein catheters placed either through the subclavian veins or the internal jugular veins.

Similarly, intravenous catheters may be used to infuse large volumes over longer periods of times with little chance of thrombosis.

Proper placement of central venous pressure monitors is necessary for accurate measurements. Ideally the catheter tip should lie between the most proximal venous valves of the subclavian or jugular veins and the right atrium.

Usually the last valve in the subclavian vein is at the level of the anterior portion of the first rib. Therefore, the tip should be medial to this point. (2.5 cm from where they join to form the brachiocephalic vein).

The most common locations for malpositioned catheters include the internal jugular vein, right atrium, and right ventricle. Arrhythmias or cardiac perforations may result from placement of lines within the heart.

Complications of central line placement may result in pneumothorax, occurring in as many as 6% of cases.

95
Q

🦢 Swan-Ganz catheters (pulmonary capillary wedge pressure monitors)

A

Used to measure pulmonary wedge pressures. These catheters allow the intensivist to have an accurate measurement of the patient’s volume status and can help differentiate between cardiac and non-cardiac pulmonary edema.

Pulmonary capillary wedge pressure catheters (PCWP) are introduced percutaneously into the venous system. They are advanced through the right heart and into the pulmonary artery. A balloon at the end of the catheter is then inflated causing the tip of the catheter to be wedged into a branch of the pulmonary artery. The tip is “floated” to a distal pulmonary artery and wedged there. Once the tip is wedged, the balloon should be deflated. Once a reading is obtained, the tip is pulled back to the main pulmonary artery. The catheter tip should ideally be positioned no more distally than the proximal interlobar pulmonary arteries. A good rule of thumb is that the catheter tip should be within the mediastinal shadow. Placement more distally increases the chance of pulmonary infarction or vessel rupture.

Malpositioning of PCWP catheters is exceedingly common, found in approximately 25% of catheters placed. This may lead to false readings and an increased risk for complications. Complications of PCWP catheter placement include pneumothorax, pulmonary infarction, cardiac arrhythmias, pulmonary artery perforation, endocarditis, and sepsis.

96
Q

Intraaortic counterpulsation balloon pump (IACB)

A

Used to decrease afterload and increase cardiac perfusion in patients with cardiogenic shock. The IACB is synchronized with either the aortic pressures or the patient’s EKG to inflate during diastole and deflate during systole. Generally, the IACB is introduced percutaneously through the right femoral artery. Proper positioning of the IACB is critical to prevent occlusion of major vessels. Ideally the catheter should be in the region of the aortic isthmus or left main bronchus and above the origins of the celiac trunk and superior mesenteric artery. During systole the balloon may appear as a fusiform air (helium) containing radiolucency.

97
Q

Transvenous Pacing Device

A

Patients in the ICU with bradyarrhythmias or heart block may require cardiac pacing. Transvenous pacers are introduced through the cephalic or subclavian vein into the apex of the right ventricle. Frontal and lateral projections are required to evaluate pacemaker placement. In the frontal view, the pacer tip should be at the apex with no sharp angulations throughout its length. On the lateral view, the tip should be imbedded within the cardiac trabeculae in such a way that it appears 3 to 4 mm beneath the epicardial fat stripe. A tip which appears beyond the epicardial fat stripe may have perforated the myocardium. Pacers placed within the coronary sinus will appear to be directed posteriorly on the lateral chest x-ray. The integrity of the pacer wire should be inspected along its entire length.

98
Q

Thoracostomy tubes

A

Placed into the pleural space to evacuate either air or fluid. In the supine patient air collects anteriorly and fluid collects posteriorly. This dictates the proper positioning of the tube.

Thoracostomy tubes placed within fissures often cease to function when the lung surfaces become opposed. Also, incorrectly placed tubes for empyemas may delay drainage and result in loculation of the purulent fluid.

In order for thoracostomy tubes to function properly all of the fenestrations in the tube must be within the thoracic cavity. The last side-hole in a thoracostomy tube is indicated by a gap in the radiopaque line. If this interruption in the radiopaque line is not within the thoracic cavity or there is evidence of subcutaneous air, then the tube may not have been completely inserted.

99
Q

Atelectasis

A

Atelectasis is a term used to describe reduced inflation in part of the lung.

Atelectasis in ICU patients occurs most frequently in the left lower lobe, presumably due to compression of the lower lobe bronchus by the heart, in the supine patient. Contributing to this tendency is the relatively greater difficulty of blind suctioning of the left lower lobe.

The etiology of atelectasis includes any process which reduces aveolar ventilation including general anesthesia, splinting from pain following surgery, or bronchial obstruction by mucus plugging. General anesthesia and surgical manipulation lead to atelectasis by causing diaphragmatic immobilization. Atelectasis is usually basilar. On physical examination, shift of the trachea to the affected side suggests volume loss.

Mobilization of secretions may be inhibited by inflammatory lung disease, edema, or tracheal intubation. The final result is reduced alveolar distention resulting in decreased surfactant production which propagates the ateletasis further. Usually atelectasis is more extensive than is suggested by the radiograph. Extensive alveolar hypoventilation may result in an effective right to left shunt and subsequent hypoxia. Atelectasis is reversible and preventable with the use of hyperventilation and incentive spirometry especially in the post-operative period.

Is most often caused by an endobronchial lesion, such as mucus plug or tumor. It can also be caused by extrinsic compression centrally by a mass such as lymph nodes or peripheral compression by pleural effusion. An unusual type of atelectasis is cicatricial and is secondary to scarring, TB, or status post radiation.

Atelectasis is almost always associated with a linear increased density on chest x-ray. The apex tends to be at the hilum. The density is associated with volume loss. Some indirect signs of volume loss include vascular crowding or fissural, tracheal, or mediastinal shift, towards the collapse. There may be compensatory hyperinflation of adjacent lobes, or hilar elevation (upper lobe collapse) or depression (lower lobe collapse). Segmental and subsegmental collapse may show linear, curvilinear, wedge shaped opacities. This is most often associated with post-op patients and those with massive hepatosplenomegaly or ascites .

Radiographically, atelectasis may vary from complete lung collapse to relatively normal-appearing lungs. For example, acute mucus plugging may cause only a slight diffuse reduction in lobar or lung volume without visible opacity. Nevertheless, the physiologic effects can be significant. In the so called mucus plugging syndrome, the association of sudden hypoxia with a normal or quasi-normal chest radiograph can lead to the suspicion of a pulmonary embolus. Mild atelectasis usually takes the form of minimal basilar shadowing or linear streaks (subsegmental or “discoid” atelectasis) and may not be physiologically significant. Atelectasis may also appear similar to pulmonary consolidation (dense opacification of all or a portion of a lung due to filling of air spaces by abnormal material), making it difficult to distinguish from pneumonia or other causes of consolidation. The distinction between atelectasis and other causes of consolidation is important, and certain clues exist to aid in making that determination. Atelectasis will often respond to increased ventilation, while pneumonia, for example, will not. Crowding of vessels, shifting of structures such as interlobar fissures towards areas of lung volume loss and elevation of the hemidiaphragm suggests atelectasis. Another key for distinguishing between atelectasis and consolidation is recognition of the typical patterns that each pulmonary lobe follows when collapsing.

100
Q

Right Upper Lobe Atelectasis

A

Right upper lobe atelectasis is easily detected as the lobe migrates superomedially toward the apex and mediastinum. The minor fissure elevates and the inferior border of the collapsed lobe is a well demarcated curvilinear border arcing from the hilum towards the apex with inferior concavity. Due to reactive hyperaeration of the lower lobe, the lower lobe artery will often be displaced superiorly on a frontal view.

101
Q

Left Upper Lobe Atelectasis

A

The left lung lacks a middle lobe and therefore a minor fissure, so left upper lobe atelectasis presents a different picture from that of the right upper lobe collapse. The result is predominantly anterior shift of the upper lobe in left upper lobe collapse, with loss of the left upper cardiac border. The expanded lower lobe will migrate to a location both superior and posterior to the upper lobe in order to occupy the vacated space. As the lower lobe expands, the lower lobe artery shifts superiorly. The left mainstem bronchus also rotates to a nearly horizontal position.

102
Q

Right Middle Lobe Atelectasis

A

Right middle lobe atelectasis may cause minimal changes on the frontal chest film. A loss of definition of the right heart border is the key finding. Right middle lobe collapse is usually more easily seen in the lateral view. The horizontal and lower portion of the major fissures start to approximate with increasing opacity leading to a wedge of opacity pointing to the hilum. Like other cases of atelectasis, this collapse may by confused with right middle lobe pneumonia.

103
Q

Left Lower Lobe Atelectasis

A

Atelectasis of either the right or left lower lobe presents a similar appearance. Silhouetting of the corresponding hemidiaphragm, crowding of vessels, and air bronchograms are standard, and silhouetting of descending aorta is seen on the left. It is important to remember that these findings are all nonspecific, often occuring in cases of consolidation, as well. A substantially collapsed lower lobe will usually show as a triangular opacity situated posteromedially against the mediastinum.

104
Q

Right Lower Lobe Atelectasis

A

Silhouetting of the right hemidiaphragm and air bronchograms are common signs of right lower lobe atelectasis. Right lower lobe atelectasis can be distinguished from right middle lobe atelectasis by the persistance of the right heart border.

105
Q

ARDS versus Congestive Heart Failure

A

While it is not always easy, it is often possible to radiographically distinguish between pulmonary edema caused by congestive heart failure (CHF) and ARDS. Indeed, both may coexist. Although both entities may share the x-ray finding of bilateral airspace opacification or “white out”, ARDS is not associated with cardiomegaly or with cephalization of pulmonary vasculature. However, cephalization may not be visible in the midst of “white out”and CHF can exist without cardiomegaly. Both of these findings may be difficult to discern in the supine patient. The patient with ARDs could also have preexistant cardiomegaly or be fluid overloaded because of sepsis.

Features that are helpful in distinguishing CHF from ARDS include the following: While cardiogenic pulmonary edema typically begins centrally in the bilateral perihilar areas, ARDS usually causes more uniform opacification. Pleural effusions are not typical of ARDS but often present in CHF. Kerley B lines are common in CHF but not in ARDS, while air bronchograms can be found in both.

Temporally, radiographic abnormalities usually closely parallel cardiogenic pulmonary edema, while the chest radiograph in ARDS may remain unremarkable for up to twelve hours and usually stabilize after the first thirty-six hours. While radiographic findings in cardiogenic edema may clear rapidly, ARDS typically clears slowly. Unlike cardiogenic edema, which, once resolved, does not leave behind permanent pulmonary changes, a percentage of ARDS cases will result in some degree of permanent pulmonary fibrosis, characterized by increased intersitital markings depending on the severity and length of time the patient was in ARDS.

106
Q

Differential Diagnosis of Acute Cough (<3 wk):

A

Common cold or viral upper respiratory tract infection

Lower respiratory tract infections (bronchitis and pneumonia)

Bacterial sinusitis

Rhinitis caused by allergens or environmental irritants

Asthma or COPD exacerbation

Cardiogenic pulmonary edema

Aspiration or foreign body

Medication reaction (eg, ACE inhibitor)

Pulmonary embolism

Bordetella pertussis

107
Q

Differential Diagnosis of Chronic Cough (>8 weeks):

A

Chronic Cough (>8 wk)

Upper airway cough syndrome

Cough-variant asthma

GERD

Nonasthmatic eosinophilic bronchitis

Bronchiectasis

Medication reaction (eg, ACE inhibitor)

Chronic bronchitis caused by smoking

108
Q

🏮HEMOPTYSIS

A

Pulmonary:

  • Bronchitis
  • Lung cancer
  • Bronchiectasis

Cardiac:

  • Mitral stenosis/acute pulmonary edema

Infectious:

  • Tuberculosis typically has radiographic abnormalities (eg, patchy or nodular opacity, multiple nodules, cavity) involving the apical-posterior segments of the upper lobes of the lungs.
  • Lung abscess
  • Bacterial pneumonia
  • Aspergillosis

Hematologic

  • Coagulopathy

Vascular

  • Pulmonary embolism
  • Arteriovenous malformation

Systemic disease

  • Granulomatosis with polyangiitis
  • Goodpasture syndrome

Other

  • Trauma
  • Cocaine use (inhalation)

Hemoptysis is defined as any expectoration of blood originating from the lower respiratory tract, with a wide spectrum from minimal blood streaking in sputum to the presence of frank blood and/or clots. There are many causes of hemoptysis, but pulmonary airway disease (eg, chronic bronchitis, bronchogenic carcinoma, bronchiectasis) ranks as the most common.

Initial management involves establishing an adequate patent airway, maintaining adequate ventilation and gas exchange, and ensuring hemodynamic stability. The patient should be placed with the bleeding lung in the dependent position (lateral position) to avoid blood collection in the airways of the opposite lung.

Dx: Bronchoscopy is the initial procedure of choice in such patients as it can localize the bleeding site, provide suctioning ability to improve visualization, and include other therapeutic interventions (eg, balloon tamponade, electrocautery).

Cx: The greatest danger in massive hemoptysis is not exsanguination but asphyxiation due to the airway flooding with blood.

109
Q

Acute bronchitis

A

Etiology

  • Preceding respiratory illness (90% viral)

Clinical presentation

  • Cough for >5 days to 3 weeks (± purulent sputum)
  • Absent systemic findings (eg, fever, chills)
  • Wheezing or rhonchi, chest wall tenderness

Diagnosis & treatment

  • Clinical diagnosis, CXR only when pneumonia suspected
  • Symptomatic treatment (eg, NSAIDs &/or bronchodilators)
  • Antibiotics not recommended

Cough lasting >5 days is characteristic of acute bronchitis, and a viral URI is the usual cause. Sputum production occurs in roughly half of patients; the typical yellow/purulent sputum is due to epithelial sloughing and is not a sign of bacterial infection.

Small amounts of blood in the sputum can occur due to inflammation and epithelial damage. Mild dyspnea and chest wall discomfort are common, and physical examination often shows wheezing as well as crackles that clear with cough, suggesting that secretions are easily mobilized (unlike in pneumonia). Fever is not typical and, when present, should raise suspicion for bacterial pneumonia or influenza.

The illness is self-limiting (although cough and airway hypersensitivity may persist for weeks), and only symptomatic treatment (eg, nonprescription pain relievers) is indicated. Antibiotics should generally be avoided as they provide no significant benefit (including in acute bronchitis due to Mycoplasma species) and are associated with adverse effects.

110
Q

Chronic bronchitis

A

Chronic bronchitis is defined as a chronic productive cough for >3 months in 2 successive years, and cigarette smoking is the leading cause.

The most common cause for nonmassive hemoptysis (< 30 mL/d) in smokers and nonsmoking patients with a normal chest radiograph is acute or chronic bronchitis. (Massive life-threatening hemoptysis is more than 200 mL of blood in 24 hours.)

Dx: Upon clinical diagnosis, office spirometry is necessary to make the diagnosis, assess the disease severity, and monitor response to treatment.

111
Q

🍒Bronchiectasis

A

Signs & symptoms

  • Cough with daily mucopurulent sputum production
  • Rhinosinusitis, dyspnea, hemoptysis
  • Crackles, wheezing

Pathophysiology

  • Infectious insult with impaired clearance

Etiologies

  • Airway obstruction (eg, cancer)
  • Rheumatic disease (eg, RA, Sjögren), toxic inhalation
  • Chronic or prior infection (eg, aspergillosis, mycobacteria)
  • Immunodeficiency (eg, hypogammaglobulinemia)
  • Congenital (eg, CF, alpha-1-antitrypsin deficiency)

Evaluation

  • HRCT scan of the chest (needed for initial diagnosis)
  • Immunoglobulin quantification
  • CF testing, sputum culture (bacteria, fungi & mycobacteria)
  • Pulmonary function testing

Bronchiectasis is an acquired disease that causes abnormal dilatation of the bronchi leading to pooling of secretions in the airways and recurrent infections. Patients typically present with cough productive of purulent sputum, shortness of breath, and pleuritic chest pain.

Lung auscultation may be normal or remarkable for wheezes, rhonchi, or crackles at the lung bases. Chest radiograph may be normal, but occasionally the damaged, dilated airways will appear as tram tracks or ring shadows. Bronchiectasis may be a sequela of foreign body aspiration, cystic fibrosis, rheumatic diseases (rheumatoid arthritis, SjÖgren disease), recurrent pulmonary infections (tuberculosis, pertussis, Mycoplasma), AIDS, or allergic bronchopulmonary aspergillosis (ABPA).

Development of the disease requires an infectious insult in combination with impaired bacterial clearance (eg, impaired immune defenses, structural airway defect). Etiologies include congenital disease (eg, cystic fibrosis, alpha-1-antitrypsin deficiency), hypogammaglobulinemia, rheumatoid arthritis, Sjögren syndrome, airway obstruction (eg, lung cancer), recurrent aspiration, and chronic or previous lung infection.

Typical symptoms of bronchiectasis include chronic cough with daily production of large amounts of mucopurulent sputum (>100 mL/day), dyspnea, sinus congestion, fatigue, weight loss, and hemoptysis. Physical examination reveals crackles and wheezing on lung auscultation. Patients may have frequent exacerbations characterized by fever, increased dyspnea, and increased sputum production. The clinical presentation is often similar to that of chronic bronchitis; however, sputum production is more prominent in bronchiectasis, and exacerbations are typically bacterial (usually viral in chronic bronchitis) and require antibiotics. In addition, although smoking is the most important risk factor for chronic bronchitis, a causal relationship with bronchiectasis has not been established.

112
Q

Bordetella pertussis

A

Symptoms in adults may be less severe than the “whooping cough” usually associated with infection in children, and a severe, persistent cough may be the primary clinical manifestation in adults. The incubation period is 7 to 10 days, which is then followed by the catarrhal phase that lasts for 1 to 2 weeks and is characterized by malaise, rhinorrhea, and mild cough. The paroxysmal phase follows in which the other symptoms improve but the cough becomes severe, often being triggered by specific activities (eg, yawning or stretching) or exposure to environmental respiratory irritants. This phase may last weeks to months if untreated followed by the convalescent phase that lasts 1 to 3 months during which the cough slowly resolves.

Tx: Pertussis is highly communicable, and treatment with a macrolide antibiotic (erythromycin for 14 days, or azithromycin for 5 days) in the first 2 weeks of infection decreases the severity of symptoms and decreases transmission of disease.

113
Q

upper airway cough syndrome (UACS); Postnasal drip

A

Upper-airway cough syndrome is caused by rhinosinus conditions including allergic, perennial nonallergic, and vasomotor rhinitis; acute nasopharyngitis; and sinusitis.

Cough is caused by mechanical stimulation of the afferent limb of the cough reflex in the upper airway in these conditions.

Symptoms of UACS include cough, nasal discharge, sensation of postnasal drip (a sensation of liquid dripping into the back of the throat), and frequent throat clearing.

Physical examination findings of UACS are cobblestoning of the posterior pharyngeal mucosa and mucoid or mucopurulent secretions at the nasopharynx or oropharynx.

Tx: Chlorpheniramine is a specific H1 antihistaminic receptor blocker that reduces the action of histamine on H1 receptors, decreasing the allergic response. In addition to blocking H1 histamine receptors, chlorpheniramine exhibits anti-inflammatory effects by blocking histamine release from mast cells and limiting the secretory response to inflammatory cytokines.

114
Q

Cough-variant asthma

A

Cough-variant asthma (in which cough is the predominant symptom) occurs in up to 57% of patients with asthma. Cough-variant asthma is present in patients who have cough as their main symptom.

Hx: The cough is typically dry and is sometimes the only symptom of asthma.

Dx: Patients with cough-variant asthma may demonstrate reversible airflow obstruction or airway hyperreactivity with bronchoprovocation testing. However, because bronchoprovocation testing may yield false-positive results, asthma should be diagnosed as a cause of chronic cough only if symptoms abate after 2 to 4 weeks of standard asthma treatment?

Tx: The treatment for cough-variant asthma is the same as for asthma in general, but the maximum symptomatic benefit may not occur for 6 to 8 weeks in cough-variant asthma.

115
Q

GERD-related cough

A

Hx: Heartburn and cough exacerbated by the recumbent position.

Tx: If lifestyle modification (weight loss, elevation of the head of the bed, avoidance of tobacco and alcohol) is unsuccessful, targeted and prolonged treatment with histamine blockers or proton pump inhibitors is recommended. Improvement in the cough may require therapy for 2 to 3 months.

116
Q

Medication reaction (eg, ACE inhibitor)

A

Approximately 15% of patients who are prescribed these medications will develop a nonproductive cough. Reported causative factors include bradykinin and substance P, which are metabolized by ACE and prostaglandins.

Smoking cessation and discontinuation of ACE inhibitors should be recommended for 4 weeks before additional evaluation.

117
Q

Malignancy

A

Risk factors that increase the risk of malignancy include male sex, age older than 40 years, a smoking history of more than 40 pack-years, and symptoms lasting for more than 1 week.

Dx: These patients should be referred for chest CT and fiberoptic bronchoscopy even if the chest radiograph is normal.

118
Q

Common cold or viral upper respiratory tract infection

A

Tx: With Mild illness and otherwise healthy = no need for treatment with an antiviral medication.

Antiviral treatment of suspected or confirmed influenza for: hospitalized patients; those with severe, complicated, or progressive illness. Other high-risk medical conditions include cardiovascular disease (except isolated hypertension), active cancer, chronic kidney disease, chronic liver disease, hemoglobinopathies, immunocompromise (including HIV disease), and neurologic diseases that impair handling of respiratory secretions.

Oseltamivir or zanamivir is indicated for those with influenza A or influenza B virus infection and for those in whom the influenza virus type is unknown.

Amantadine and rimantadine are related antiviral medications in the adamantane class that are active against influenza A viruses but not influenza B viruses. In recent years, widespread adamantane resistance among influenza A strains has been noted.

119
Q

🏊🏽‍♂️ Drowning

A

Drowning is death from suffocation after submersion. Freshwater drowning in swimming pools is actually more common than saltwater drowning. Noncardiogenic pulmonary edema is a complication of near drowning (survival after suffocation from submersion). This is a result of direct pulmonary injury, loss of surfactant, and contaminants in the water. Respiratory failure, severe hypothermia, and neurologic injury are the three most common threats to life after submersion.

120
Q

☠ Poisoning

A

Methanol (methyl alcohol or wood alcohol) is found in “moonshine”; patients present with blindness.

Ethylene glycol is found in antifreeze; ingestion leads to seizures and coma. Patients develop oxalate crystals in the urine, and deposition may result in renal failure. Both methanol and ethylene glycol overdoses are treated with ethanol to prevent the formation of formic acid (toxic).

Carbon monoxide Appear cherry red but are hypoxemic.

Cyanide poisoning is associated with a bitter almond odor elevated venous oxygen saturation because tissues fail to take up arterial oxygen.

Diabetic ketoacidosis (DKA) fruity breath odor. Hyperpnea refers to respiration, which is deep (increase in tidal volume) as well as rapid (Kussmaul breathing).

Arsenic ingestion and parathion poisoning are associated with a garlic odor.

Hydrogen sulfide mercaptans: Rotten egg odor

Naphthalene (mothballs): The odor of camphor, along with abdominal pain and hemolysis

121
Q

Carbon monoxide poisoning

A

Epidemiology

  • Smoke inhalation
  • Defective heating systems
  • Gas motors operating in poorly ventilated areas

Manifestations

  • Mild-moderate
    • Headache, confusion
    • Malaise, dizziness, nausea
  • Severe
    • Seizure, syncope, coma
    • Myocardial ischemia, arrhythmias

Diagnosis

  • ABG:carboxyhemoglobin level
  • ECG ± cardiac enzymes

Treatment

  • High-flow 100% oxygen
  • Intubation/hyperbaric oxygen (severe)

CO is a byproduct of combusting organic matter (eg, oil, gas, wood). Exposure to toxic levels is more likely in enclosed or poorly ventilated areas. CO tightly binds hemoglobin - forming carboxyhemoglobin - with an affinity much greater than that of oxygen. Nonsmokers have low levels (<3%) of carboxyhemoglobin (due to normal enzymatic reactions). Cigarette smokers may have carboxyhemoglobin levels as high as 10%. Even though patients at this level are generally asymptomatic, small additional amounts of CO exposure may cause toxicity manifesting as headache, malaise, and nausea.

Carboxyhemoglobin shifts the oxygen dissociation curve to the left, impairing the ability of heme to unload oxygen at the tissue level. This results in tissue hypoxia. The kidney responds to tissue hypoxia by producing more erythropoietin (EPO). EPO stimulates the bone marrow to differentiate more red blood cells. Chronic CO toxicity is a cause of secondary polycythemia.

Pulse oximetry does not differentiate between carboxyhemoglobin and oxyhemoglobin; it cannot be used in the diagnosis of CO poisoning. Diagnosis is made by arterial blood gas with cooximetry.

122
Q

Pulmonary contusion

A

Clinical features

  • Present <24 hours after blunt thoracic trauma
  • Tachypnea, tachycardia, hypoxia

Diagnosis

  • Rales or decreased breath sounds
  • CT scan (most sensitive) or CXR with patchy, alveolar infiltrate not restricted by anatomical borders

Management

  • Pain control
  • Pulmonary hygiene (eg, incentive spirometry, chest PT)
  • Supplemental oxygen & ventilatory support

Blunt force to the chest wall can injure the underlying lung, resulting in alveolar hemorrhage and edema. Subsequent resuscitative fluid administration can exacerbate the alveolar edema, leading to progressive dyspnea, tachypnea, and hypoxemia. On CT scan, the alveolar edema can appear as ground-glass opacities in the lung adjacent to the affected chest wall (eg, anterior, peripheral lung in this patient); this pattern and distribution is classic for pulmonary contusion.

Pulmonary contusion is common (25%-35% of cases) following blunt thoracic trauma. Because clinically significant alveolar edema may take up to 24 hours to accumulate, pulmonary contusion may not be apparent immediately following an injury, and initial chest x-ray is often negative. CT scan of the chest is more sensitive and can identify the classic irregular, nonlobular (ie, not restricted by anatomic landmarks) infiltrates. Management involves pain control, pulmonary hygiene, and respiratory support.

123
Q

Airway deterioration

A

Severe hypoxemia (eg, PaO2 <60 mm Hg on room air)

In patients unable to maintain adequate oxygen saturations, bag-valve-mask ventilation (BVM) with 100% oxygen (to keep oxygen saturation ≥ 88%) should be initiated. If BVM does not result in adequate oxygenation (ie, oxygen saturation remains low, as in this patient), endotracheal intubation using a video laryngoscope (to facilitate direct visualization of the epiglottis) should be attempted.

However, given the risk of rapid respiratory deterioration, failure of a single attempt at endotracheal intubation with a video laryngoscope should immediately prompt the establishment of a surgical cricothyrotomy by the most experienced provider available (preferably an otolaryngologist or general surgeon). Cricothyrotomy establishes an airway below the epiglottal swelling and potential obstruction.

124
Q

Mediastinal Mass

A

Anterior:

Consist of the 4 “T’s” (Terrible lymphadenopathy, Thymic tumors, Teratoma, Thyroid mass) and aortic aneurysm, pericardial cyst, epicardial fat pad.

Usually CT or fine needle aspiration is needed to make the definitive diagnosis of an anterior mediastinal mass.

Middle:

Common cause is lymphadenopathy due to metastases or primary tumor. Other causes include hiatial hernia, aortic aneurysm, thyroid mass, duplication cyst, bronchogenic cyst, vascular masses, and pleuropericardial cysts.

Posterior:

Is the likely area for neurogenic tumors, lymphomas, pheochromocytomas, myelomas, meningoceles, meningomyeloceles, gastroenteric cysts, and diverticula.

125
Q

🍇 PULM

A
126
Q

Bacterial tracheitis

A

Bacterial tracheitis presents with fever, stridor, and respiratory distress.

Croup presents with a “barky” cough, hoarseness, stridor, and fever. In both conditions, onset is gradual (over days), and neck x-ray (posterioanterior view) reveals subglottic narrowing (eg, steeple sign) and a normal epiglottis

127
Q

Epiglottitis

A

Epiglottitis is a rare, potentially fatal infection that presents with acute onset of fever, sore throat, and signs of upper airway obstruction (eg, stridor, drooling).

Severe sore throat with a benign-appearing oropharynx. Adults may have dyspnea, drooling, and stridor. Obtain urgent otolaryngology consultation, and do not attempt to examine the throat. A lateral neck film may show an enlarged epiglottis (“thumbprint sign”). Because of
the possibility of impending airway obstruction, the patient should be admitted to an intensive care unit for close monitoring. The most likely organism causing this infection is H influenzae.

X-ray is not required for diagnosis if clinical suspicion is high, but lateral view shows an enlarged epiglottis, suggestive of edema. Diagnosis is confirmed via direct visualization of an edematous epiglottis. However, detailed oropharyngeal examination is often deferred in children due to risk of laryngospasm from provoked aggravation. Direct laryngoscopy during intubation (a controlled setting to secure the airway) is often preferred for diagnosis and management.

128
Q

Laryngomalacia

A

Laryngomalacia is the most common cause of inspiratory stridor in infants. Although the precise etiology is not known, the supraglottic tissues appear floppy and collapse on inspiration to partially block the airway.

Crying and feeding worsen the stridor due to increased airflow.

Prone positioning improves the stridor because the tongue moves anteriorly, partially relieving the obstruction.

Patients typically have an omega-shaped (Ω) epiglottis, short aryepiglottic folds, and characteristic inspiratory collapse of the supraglottic tissues (eg, epiglottis, arytenoids), leading to partial obstruction of the airway and thus inspiratory stridor.

During inspiration, faster airflow causes decreased intraluminal pressure in the airways (eg, drop in intralaryngeal pressure). In patients with a more collapsible extrathoracic airway (eg, laryngomalacia), the decreased pressure beyond the larynx leads to increased airway narrowing, resulting in turbulent flow and inspiratory stridor. Any increase in breathing effort (eg, feeding, crying) increases airflow, worsens supraglottic collapse, and increases stridor.

Stridor from laryngomalacia usually begins in the neonatal period and is loudest at age 4-8 months. Most patients also have concurrent symptomatic gastroesophageal reflux (eg, vomiting, arching of the back with feeds). Antireflux treatment often improves breathing symptoms (for unclear reasons). Infants who are feeding and growing well may be watched expectantly as laryngomalacia typically resolves spontaneously by age 18 months. Surgery may be needed for moderate to severe cases with respiratory distress, cyanosis, or failure to thrive.

129
Q

Tracheomalacia

A

Inspiration decreases intrathoracic pressure due to diaphragm contraction (which lowers the diaphragm). In patients with a more collapsible intrathoracic airway (eg, tracheomalacia), the decreased pressure widens the intrathoracic tracheal airway.

In contrast, expiration increases intrathoracic pressure. In patients with tracheomalacia, the increased pressure narrows the intrathoracic tracheal airway, leading to expiratory stridor.

130
Q

Retropharyngeal abscess

A

Retropharyngeal abscess presents with fever, dysphagia, drooling, stridor, and a stiff neck in young children. Examination reveals swelling of the posterior pharyngeal wall

131
Q

Bronchopulmonary dysplasia (BPD)

A

Chronic lung disease of the neonate which results from repeated insult to the neonatal lung from factors such as mechanical ventilation, prolonged oxygen exposure, and inflammation.

Risk factors for BPD include prematurity, low birth weight (<2,500 g), respiratory distress syndrome, and mechanical ventilation. Surfactant therapy does not prevent BPD development but may reduce mortality from it. Most patients with BPD improve over 2-4 months; some develop pulmonary arterial hypertension.

132
Q

Transient tachypnea of the newborn

A

Pathophysiology

  • Retained fetal lung fluid

Risk factors

  • Cesarean delivery
  • Prematurity
  • Maternal diabetes

Clinical findings

  • Tachypnea, increased work of breathing
  • Clear breath sounds
  • Chest x-ray: Hyperinflation, fluid in fissures

Management

  • Supportive care (eg, oxygen, nutrition)
  • Self-resolution in 1-3 days

Transient tachypnea of the newborn (TTN) is a condition caused by delayed resorption and clearance of alveolar fluid. Normally, mature fetal lungs begin to reabsorb liquid in late gestation in response to increased hormonal (catecholamine) signals. This resorption mechanism increases during labor. Therefore, patients born prematurely or by cesarean delivery are at increased risk of TTN.

Excess pulmonary fluid can cause respiratory distress (eg, tachypnea, grunting, retractions) and hypoxia in newborns within a few hours after delivery. However, breath sounds are often clear as fluid remains in the interstitial space rather than in the alveoli. Findings on chest x-ray (eg, hyperinflation, fluid in interlobar fissures) confirm the diagnosis, and patients are treated supportively with supplemental oxygen as needed. TTN is a self-limited condition that spontaneously resolves within a few hours to days as passive fluid resorption is completed.

133
Q

Respiratory distress syndrome (RDS)

A

A pulmonary condition caused by immature lungs and surfactant deficiency.

RDS incidence is inversely proportional to gestational age. The most important risk factor for RDS is prematurity; other factors that increase RDS risk include male sex, perinatal asphyxia, maternal diabetes, and cesarean section without labor. Maternal diabetes increases the incidence of RDS by delaying the maturation of pulmonary surfactant production. Maternal hyperglycemia causes fetal hyperglycemia, which in turn triggers fetal hyperinsulinism. High levels of circulating insulin antagonize cortisol and block the maturation of sphingomyelin, a vital component of surfactant.

RDS presents with tachypnea, retractions, grunting, nasal flaring, and cyanosis at birth. Despite initial resuscitation and respiratory support, patients can continue to decompensate and require intubation. Chest radiograph shows a diffuse reticulogranular pattern (“ground-glass opacities”) and air bronchograms. Treatment consists of antenatal prevention with corticosteroidsand postnatal treatment with exogenous surfactant and respiratory support.

134
Q

Bronchiolitis (RSV)

A

Epidemiology

  • Age <2 years
  • RSV most common cause

Clinical presentation

  • Antecedent nasal congestion/discharge & cough
  • Wheezing/crackles & respiratory distress (eg, tachypnea, retractions, nasal flaring)

Treatment

  • Supportive care

Prevention

  • Palivizumab for infants with the following conditions:
    • Preterm birth <29 weeks gestation
    • Chronic lung disease of prematurity
    • Hemodynamically significant congenital heart disease

Complications

  • Apnea (especially infants age <2 months)
  • Respiratory failure

Bronchiolitis is a common winter respiratory tract infection caused primarily by respiratory syncytial virus (RSV). In older children, RSV infection is generally a self-limiting, mild, upper respiratory tract infection (eg, nasal congestion, rhinorrhea). However, children age <2 years tend to have lower respiratory tract involvement. The wheezing and/or crackles and respiratory distress can have a waxing/waning course that peaks on days 5-7 of illness. Diagnosis is primarily clinical, and treatment is largely supportive (eg, hydration, saline nasal drops, nasal bulb suction). Palivizumabis a monoclonal antibody against RSV that is used for prophylaxis in children age <2 years who are at exceptionally high risk of complications (Table).

Cx: Infants age <2 months are at high risk of developing apnea and respiratory failure from bronchiolitis. In addition, they tend to develop recurrent wheezing throughout childhood.

135
Q

Foreign body aspiration

A

Clinical features

  • Sudden-onset cough, dyspnea
  • Cyanosis
  • ± History of choking episode

Examination findings

  • Wheezing and/or stridor
  • Focal area of diminished breath sounds

X-ray findings

  • Hyperinflation of affected side
  • Mediastinal shift toward unaffected side
  • Atelectasis if obstruction is complete
  • ± Foreign body

Management

  • Rigid bronchoscopy

FB aspiration is most common in children age 1-3 and usually involves foods (eg, nuts, seeds) or small toys. Patients typically have acute-onset cough, dyspnea, and stridor. Wheezing may be focal or generalized, and decreased breath sounds on the affected side are characteristic. There is often a history of a preceding choking episode; however, FB aspiration should always be considered in a pediatric patient with a sudden onset of unexplained symptoms.

Over half of aspirated FBs lodge in the right mainstem bronchus. X-ray findings in partial FB obstruction include hyperinflation distal to the obstruction (due to air trapping during expiration) and mediastinal shift away from the affected side. Complete obstruction of one bronchus can result in ipsilateral atelectasis and, over time, postobstructive pneumonia and/or localized bronchiectasis. Most aspirated objects are radiolucent and not identified on x-ray.

When FB aspiration is suspected, immediate rigid bronchoscopy is performed to confirm the diagnosis and remove the aspirated object.

136
Q

Laryngeal papillomatosis

A

Laryngeal papillomas are caused by human papillomavirus (HPV) subtypes 6 and 11, which are also the subtypes most likely to cause genital warts (condyloma acuminatum), and therefore most likely acquired via vertical transmission prior to delivery (because neither vaginal nor cesarean delivery prevent transmission).

Although benign, RRP is associated with significant morbidity (eg, voice problems, airway obstruction, repeated operative interventions). In addition, the clinical course is variable and unpredictable with fluctuations in severity. In rare instances, it can spread beyond the vocal cords to involve the lower respiratory tract. Medical therapy (eg, interferon, cidofovir) has limited efficacy; therefore, the mainstay of treatment remains surgical debridement, and patients often require many procedures. The incidence of RRP in children is decreasing, likely due to increased rates of maternal HPV vaccination.

137
Q

Flail chest

A

Flail chest occurs when fracture of ≥3 contiguous ribs in ≥2 locations creates an isolated chest wall segment (flail segment) that moves paradoxically (opposite) to the rest of the rib cage during respiration. Typically, the extreme trauma to the chest wall that creates the flail segment also contuses the underlying lung (eg, causing diminished anterior breath sounds beneath this patient’s anterior flail segment).

Flail chest negatively impacts respiration and oxygenation in multiple ways, including the following:

  • Impaired generation of negative intrathoracic pressure during inspiration and increased dead space during expiration cause ineffective ventilation.
  • Pulmonary contusion (with alveolar hemorrhage and edema) in the underlying lung impedes oxygen diffusion.
  • Fracture-related pain causes respiratory splinting and atelectasis.

Flail chest often results in respiratory failure requiring mechanical positive pressure ventilation, which, due to positive pressure, can force the flail segment to move outward with the rest of the rib cage during inspiration.

138
Q

RIB fractures

A

Rib fracture location & associated injuries

Ribs 1-3

  • Subclavian vessels, brachial plexus, mediastinal vessels (eg, aorta)

Ribs 3-6

  • Cardiovascular

Ribs 9-12

  • Intraabdominal: liver (right), spleen (left), kidney (posterior ribs 11 & 12)

Rib fractures can cause many life-threatening injuries. Fractures of ribs 9-12 can injure intraabdominal organs (eg, liver, spleen, kidney). Unlike injury to a hollow viscus, which may show free air under the diaphragm on upright x-ray, injuries to solid organs such as the liver or spleen are typically not visible on plain abdominal x-ray. Therefore, suspicion of intraabdominal organ injury should prompt CT scan of the abdomen, preferably with IV contrast to better visualize solid organ injury and potentially detect contrast “blush” (extravasation) at the site of bleeding.

Any level

  • Pulmonary
139
Q

FV Loops

A

fixed upper-airway obstruction: The obstruction limits airflow during inspiration and expiration, causing flattening of the top and bottom of the flow-volume curve.

Asthma causes intrapulmonary airway obstruction via bronchoconstriction. This decreases airflow during the effort-independent phase of exhalation, causing flow-volume loops to have a “scooped-out” pattern during exhalation.

Pneumothorax decreases lung ventilation by preventing complete expansion of the affected lung. Pulmonary edema will also decrease lung ventilation by reducing pulmonary compliance. These conditions cause a restrictive pattern that is characterized by decreased lung volumes with expiratory flow rates that are increased (right shift) relative to the low lung volumes.

140
Q

😷 Anesthesia

A
141
Q

Mechanical ventilation

A

Invasive mechanical ventilation may be required in hypercapnic patients with poor mental status (eg, somnolence, lack of cooperation, inability to clear secretions), hemodynamic instability, or profound acidemia (pH <7.1).

Mechanical ventilation improves oxygenation by providing an increased fraction of inspired oxygen (FiO2) and positive end-expiratory pressure (PEEP) to prevent alveolar collapse.

FiO2: Although there is no strict cutoff FiO2 value for oxygen toxicity, FiO2 levels <60% are considered generally safe. Immediately following intubation, a high FiO2 (eg, >60%, or 0.6) is usually provided, and ventilator settings can subsequently be adjusted based on the results of the first arterial blood gas analysis. Although it could be increased further, FiO2 is usually weaned to <60% as quickly as possible.

Cx: Prolonged high FiO2 can cause oxygen toxicity as it can lead to the formation of proinflammatory oxygen free radicals and predispose to atelectasis as alveolar nitrogen is displaced, resulting in worsened oxygenation.

PEEP: prevents alveolar collapse during respratopry cycles and may also reopen some alveili. that have already collapsed, reducing shunting. High PEEP approaches may improve outcomes in patients with ARDS.

PaO2: An important measure of oxygenation, is influenced mainly by FiO2 and PEEP. The goal is to maintain arterial partial pressure of oxygen (PaO2) at 55-80 mm Hg, which roughly corresponds to oxygen saturations >88%-95%.

PaCO2: The arterial partial pressure of carbon dioxide is a measure of pulmonary minute ventilation, is affected mainly by the respiratory rate (RR) and tidal volume (TV).

Assist control mode: Delivers a predetermined tidal volume with every breath. Inspiratory cycles can be initiated by the patient, but if the patient fails to breathe at a set minimum rate, then the ventilator will deliver the tidal volume on its own.

Tidal volumes should be about 6 ml/kg of ideal body weight.

142
Q

Delayed Emergence

A

Return to consciousness after anesthesia (emergence) typically occurs within 15 minutes of extubation; at a minimum, patients should be responsive with intact protective (eg, gag) reflexes within 30-60 minutes of the last administration of an anesthetic or adjuvant agent (eg, opiate, muscle relaxant). Delayed emergence occurs when a patient fails to regain consciousness within the expected window. The etiology is typically multifactorial but generally occurs due to 1 of 3 major causes:

  • Drug effect: Preoperative drug ingestion (eg, opiates, benzodiazepines, illicit drugs, anticholinergic drugs, antihistamines) may potentiate anesthetic effects. Prolonged anesthesia duration or higher medication doses may also delay emergence.
  • Metabolic disorder: Common etiologies include hyper- or hypoglycemia, hyper- or hypothermia, hyponatremia, and liver disease.
  • Neurologic disorder: Intraoperative stroke, seizure (or postictal state), or elevation of intracranial pressure can cause prolonged alterations in mental status.

Management of acute respiratory failure includes ventilatory support (eg, bag and mask, reintubation); reversal agents (eg, naloxone) may also be indicated.

143
Q

Malignant hyperthermia

A

Epidemiology

  • Genetic mutation alters control of intracellular calcium
  • Triggered by volatile anesthetics, succinylcholine, excessive heat

Manifestations

  • Masseter muscle/generalized rigidity
  • Sinus tachycardia
  • Hypercarbia resistant to increased minute ventilation
  • Rhabdomyolysis
  • Hyperkalemia
  • Hyperthermia (⌛late manifestation)

Treatment

  • Respiratory/ventilatory support
  • Immediate cessation of causative anesthetic
  • Dantrolene

MH is an autosomal dominant or sporadic skeletal muscle receptor disorder marked by excessive calcium release following exposure to succinylcholine or a volatile anesthetic (eg, halothane).

In MH, sustained muscle contraction leads to:

  • Hypercarbia (due to increased levels of cellular metabolism) that does not improve with increased minute ventilation (tachypnea)
  • sinus tachycardia
  • masseter/generalized muscle rigidity
  • Myoglobinuria, with dark urine (due to muscle breakdown) 🍖
  • hyperthermia (late manifestation due to the sustained contractions generating more energy than the body can dissipate; not usually present initially).

Most cases arise shortly after induction or during maintenance of anesthesia, but symptoms can occur soon after anesthetic cessation. Urgent treatment with dantrolene (a skeletal muscle relaxant) and supportive care are required to prevent death.

144
Q

Airway deterioration

A

Severe hypoxemia (eg, PaO2 <60 mm Hg on room air)

In patients unable to maintain adequate oxygen saturations, bag-valve-mask ventilation (BVM) with 100% oxygen (to keep oxygen saturation ≥ 88%) should be initiated. If BVM does not result in adequate oxygenation (ie, oxygen saturation remains low, as in this patient), endotracheal intubation using a video laryngoscope (to facilitate direct visualization of the epiglottis) should be attempted.

However, given the risk of rapid respiratory deterioration, failure of a single attempt at endotracheal intubation with a video laryngoscope (as in this patient) should immediately prompt the establishment of a surgical cricothyrotomy by the most experienced provider available (preferably an otolaryngologist or general surgeon). Cricothyrotomy establishes an airway below the epiglottal swelling and potential obstruction.

145
Q

Pheochromocytomas and paraganglionomas

A

Pheochromocytomas and paraganglionomas are catecholamine-producing tumors arising from chromaffin cells of the adrenal medulla or extra-adrenal paraganglia, respectively.

Hypertension in pheochromocytoma can be intermittent or sustained. Paroxysms of severe hypertensioncan be precipitated by increases in intra-abdominal pressure (eg, tumor palpation, positional changes), surgical procedures, and a number of medications, particularly anesthetic agents. In addition, nonselective beta blockers can cause a state of unopposed alpha adrenergic stimulation leading to vasoconstriction and paradoxical hypertension. For this reason, alpha adrenergic blockers (eg, phenoxybenzamine) should be administered prior to beta blockers in patients with pheochromocytoma.

146
Q

Airway Pressure

A

Measurement of airway pressures can be useful in mechanically ventilated patients. The peak airway pressure (the maximum pressure measured as the tidal volume is being delivered) equals the sum of the resistive pressure (flow x resistance) and the plateau pressure.

Peak airway pressure = resistive pressure + plateau pressure

The plateau pressure is the pressure measured during an inspiratory hold maneuver, when pulmonary airflow and thus resistive pressure are both 0. It represents the sum of the elastic pressure and positive end-expiratory pressure (PEEP).

Plateau pressure = elastic pressure + PEEP

Elastic pressure is the product of the lung’s elastance and the volume of gas delivered. Because elastic recoil is inversely related to lung compliance, the elastic pressure can be calculated as tidal volume/compliance. Decreased compliance (eg, pulmonary fibrosis) causes stiffer lungs and higher elastic pressure.

147
Q

Mediastinal Mass

A

Anterior:

Consist of the 4 “T’s” (Terrible lymphadenopathy, Thymic tumors, Teratoma, Thyroid mass) and aortic aneurysm, pericardial cyst, epicardial fat pad. Usually CT or fine needle aspiration is needed to make the definitive diagnosis of an anterior mediastinal mass.

Four Ts = Thymomas, Teratomas, Thyroid masses, and para Thyroid masses.

Other anterior mediastinal masses include: retrosternal thyroid, or nonseminomatous germ cell tumor. Primary mediastinal germ cell tumors occur predominantly in young male patients and are locally invasive. β-hCG and AFP are typically elevated in nonseminomatous germ cell tumors.

If the mass is large, patients may complain of chest heaviness or discomfort. Hoarseness, Horner’s syndrome, and facial and upper extremity edema may occur when the tumors invade locally.

Middle:

Common cause is lymphadenopathy due to metastases or primary tumor. Other causes include hiatial hernia, aortic aneurysm, thyroid mass, duplication cyst, and bronchogenic cyst (benign). Also consider lymphomas, vascular masses, pericardial cysts, or tracheal tumors.

Posterior:

Neurogenic tumors are located in the posterior mediastinum. These include: meningocele, enteric cysts, lymphomas, pheochromocytomas, diaphragmatic hernias, myelomas, meningomyeloceles, meningoceles, gastroenteric cysts, and diverticula, esophageal tumors; leiomyomas, and aortic aneurysms. MRI is the best modality to evaluate posterior mediastinal masses.

148
Q

Mediastinitis

A

Mediastinitis can complicate up to 5% of sternotomies. Patients typically present post-operatively (usually within 14 days) with fever, tachycardia, chest pain, leukocytosis, and sternal wound drainage or purulent discharge.

Chest x-ray usually shows a widened mediastinum in non-postoperative mediastinitis, but this can also be seen in postoperative mediastinitis after cardiac surgery. The diagnosis is usually clinically made and confirmed during surgery when pus is noted in the mediastinum.

Tx: Postoperative mediastinitis requires drainage, surgical debridement with immediate closure, and prolonged antibiotic therapy. Antibiotics alone do not appropriately treat mediastinitis. Acute mediastinitis has a mortality rate of 10%–50%, even with appropriate treatment.

149
Q

Pulmonary Function Tests

A

TLC Total lung capacity (volume of gas in lungs at the end of maximal inspiration)

FRC Functional residual capacity (volume of gas in the lungs at relaxation, when the inward pull of lungs is balanced by the outward pull of the relaxed chest wall)

RV Residual volume (FRC − ERV, volume of gas left in lungs after maximal exhalation)

ERV Expiratory reserve volume (volume of gas expired between FRC and RV)

Expiratory flow can be measured at specific portions of exhaled vital capacity, termed forced expiratory flow (FEF), followed by a number to represent the percentage of the FVC at which the flow was measured (e.g., FEF 75 , FEF 50 , FEF 25 ). Expiratory flow can also be measured over a volume range (e.g., FEF 25-75 ).

Obesity and asthma are the most likely causes of an increased Dlco.

Left to right shunt, which is a rare cause of an increased Dlco.

An isolated reduction in Dlco is most often associated with emphysema or fibrosis